Sunteți pe pagina 1din 50

P hysi cs | 22.

27

Solved Examples

JEE Main/Boards d ΦB E
Using formula E = , I= & dq = I × dt
dt R
Example1:A coil made up of inductance L=50 µH and
resistance r=0.2 Ω is connected to a battery of e.m.f.=5.0
∆φ 8 × 10−3
When ∆t=0.01 s − B =
E= 0.8 V
=
V.A resistance R=10 Ω is connected parallel to the coil. ∆t 0.01
Now at some instant the connection of the battery is
E 0.8
switched off. Find the amount of heat generated in the I
= = = 0.16 A
coil after switching off the battery. R 5
& ∆q = I × ∆t = 0.16 × 0.01 = 1.6 × 10−3 C
Sol: In LR circuit, the magnetic energy is stored in
When ∆t=0.01 s
1
inductor and is U=L L × I2
2 ∆φ 8.0 × 10−3
Given: (i) L= 50 µH , (ii) r=0.2 Ω, − B =
E = 0.4 V
=
∆t 0.02
(iii) R=10 Ω
E 0.4
We want to find the fraction of energy lost by the I
= = = 0.8 A
inductor in the form of heat. R 5
& ∆q = I × ∆t = ( 0.08 )( 0.02 ) = 1.6 × 10−3 C
Total energy stored in the inductor is
2
1 2 1 V Example 3: A coil of area 2 m2 is placed in magnetic
UL =
= Li L 
2 0 2 r fieldwhich varies as=
B ( )
2t2 + 2 T with area vector in
the direction of B. What is the magnitudeof E.M.F.at
∴ Fraction of energy lost across inductor as heat
t=2s?
r LV 2 50 × 10 −6 × 52
= UL •= = =3.1×10−4J
(R + r ) 2r (R + r ) 2 × 0.2 (10 + 0.2) Sol: The rate of change of magnetic flux linked with the
= 3.1 × 10 J −4 dφ
coil is equal to the induced e.m.f. in the coil E = −
Example 2: A square loop ACDE of area 20 cm2 and dt
resistance 5 Ω is rotated in a magnetic field B=2T
through 1800
Find the magnitude of E, i and ∆q after time
(a) 0.01s and (b) in 0.02s.

Sol: When the loop is rotated in external magnetic


field, the change in flux linked with the loop induces
e.m.f. in it.
 
Let S be the area

vector of loop. Before rotation
 S is
in direction to B . After

rotating loop by 180o S is in
opposite direction to B . We want to find E.M.F. through the coil when

Hence, flux through the loop before rotation is t=2 s. If we find the rate of change of flux, we have
E.M.F.
φi = BS cos00 = 2 × 20 × 10−4 =4.0×10−3 Wb  ... (i)
For θ =0= , φ BA =
cos θ BA cos 0
o
=
& flux × 10−3 Wb
4.0passing through the loop...(1)
when it is rotated by dφ dB
Differentiating the above equation, we get = .A
180 ,
0
dt dt
φf =BS cos1800 =− 1 × 2 × 20 × 10−4 =4.0×10−3 Wb  ... (i) dB  dΦB 
E A. = A ( 4t + 4 )
⇒=  |=
E| 
−3 dt  dt 
= −4.0 × 10
Therefore, changeWbin flux, ...(2)
for =
A 2; | E=| 8t + 8
∆φB = φf − φi = − 8.0 × 10−3 Wb
When t=2 s, |E|=16+8=24 V
2 2 . 2 8 | Electromagnetic Induction and Electromagnetic Waves

Example 4: A current i=(3+2t)×10-2 Aincreases at a Example 5: What inductance would be needed to store
steady rate in a long straight wire. A small circular loop 1.0kWh of energy in a coil carrying a 200Acurrent?
of radius 10-3 m has its plane parallel to the wire and
placed at a distance of 1m from the wire. The resistance
(1kWh= 3.6 × 106 J )
of the loop is 8 mΩ. Find the magnitude and the 2U
Sol: The inductance in the coil is L =
direction of the induced current in the loop. i2
Given: (i) energy stored in inductor UL=1 kWh=3.6 MJ,
Sol: As the circular loop is small, the magnetic field (ii) Current =200 A.
through it can be assumed to be uniform, having
We want to find inductance of coil.
magnitude equal to that of the field at the center of the
circular loop, and flux associated with loop is φ = Bπr 2 . 1
The energy stored in inductor is UL = Li2
dφ 2
The emf induced in loop is E = .
dt The inductance is
1m 2U 2 × 3.6 × 106
∴L= = = 180 H
i2 ( 200 )
2

Example 6: The two rails of a railway track insulated


from each other and the ground are connected to a
millivolt-meter. What is the reading of the voltmeter
Wire Loop when a train travels at a speed of 108 kmh-1 along the
track? Given the vertical component of earth’s magnetic
field = 2 × 10−4 T& separation between the rails= 1m.
The arrangement is shown in Figure. The field due to
straight wire at the center of loop is: Sol: Here the train can be considered to move
µ 2I 2I perpendicular to the earth’s magnetic field. Due to
B= 0 =10 −7 × = 2I × 10−7 T
4π d 1 motion of the train, motional e.m.f. is induced in the
& flux linked with the loop is dφ
axle of train, given by E =
− =Bv sin θ
dt
( )
2
φ = BA= B × πr 2 = 2I × 10−7 × π × 10−3 Wb The train moves in a direction perpendicular to the
component of the earth’s magnetic field. So the flux
(Area of coil is very small so B over it can be taken to associated with the axle of train changes such that the
be constant) induced E.M.F. in axle is given by
E.M.F. Einduced in the loop due to change of current is dφ
E= − = Bv sin θ  ...(1) ... (i)
dφ dI dt
e= = 2π × 10 −13   
dt dt As (v × B) is parallel to  , θ =0o

 I =( 3 + 2t ) × 10−2 ∴ E=− Blv  ...(2) ... (ii)


where l=1m, BV= 2 × 10 −4
dI
So, = 2 × 10−2 As−1
dt 180 × 1000
=&v = 50ms−1 ...(3) ... (iii)
And hence, e = 2π × 10 −13
× 2 × 10 −2
= 1.26 × 10 −14
V 60 × 60 

Induced current in the loop From (i),(ii) & (iii)

E 1.26 × 10−14 E = 2 × 10−4 × 1 × 50 = 10 × 10−3 mV


I =
= = 1.6 × 10−11 A
R 8 × 10−4 ∴ Milli-voltmeter will read 10 mV when the train passes
with a speed of 108 km/h.
Due to an increase in the current in the wire, the flux
linked with the loop will increase.So in accordance
with Lenz’s law, the direction of the current induced Example 7: A very small circular loop of area 5 cm2
in the loop will be opposite of that in the wire, i.e., &resistance 2 Ω, and negligible inductance is initially
anticlockwise. coplanar and concentric, with a much larger fixed
circular loop of radius 10cm. A constant current of 1
P hysi cs | 22.29

Ais passed in the bigger loop and the smaller loop is is observed that when the terminal velocity is attained,
rotated with angular velocity wrad/s about a diameter. the power dissipated in R1 and R2 are 0.76 W and 1.2 W
Calculate (a) the flux linked with the smaller loop (b) respectively. Find the terminal velocity of the bar and
induced e.m.f. and current in the smaller loop as a the values of R1 and R2.
function of time.
A R2 C

 FM

IA
a - I +
b
mg

B D
R1
Sol: Current in the larger loop produces magnetic field
at the center of the loop. Magnetic flux is linked with Sol: The motional e.m.f. induced in the bar is E=ℓBv. The
the smaller loop. When the smaller loop is rotated, flux direction of induced current in the bar is as shown in
linked with it changes, and thus e.m.f. is induced in it. Figure. By Fleming’s left hand rule the ampere force on
the bar will be vertically upwards.
(a) The Figure represents the arrangement of coils.
When current passes through the larger loop, the field The bar falling freely under action of gravity will acquire
at the center of larger loop is, terminal velocity only when its motion is opposed by
magnetic force FM=Bil,
µ0 I µ 0 2π × I 2π × 1 Wb
B1 = = = 10 −7 × = 2π × 10 −6 Such that Bil=mg
2R 4π R 0.1 m2
0.2 × 9.8 9.8
is normal to the area of smaller loop. i.e., I
= = A
0.6 × 1 3
The smaller loop is rotating at angular velocityω. The total power dissipated in the circuit if E is the E.M.F.
Therefore the angle of rotation is θ = ωt w.r . to B linked with the coil is
The flux linked with the smaller loop at time t,
E × I = P = P1 + P2
φ2= B1S2 cos θ= (2π × 10 )(5 × 10 ) cos(ωt)
−6 −4
⇒ E
=
( 0.76
=
+ 1.20 )
0.6 V
−9
i.e., φ2 = π × 10 cos(ωt) Wb ( 9.8 / 3)
E 0.6
The E.M.F. E=l•BvT ∴ vT = = =1 ms−1
(b) The induced e.m.f. in the smaller loop, Bl 0.6 × 1
dφ2
E2 = −
dt
d
= −
dt
(
π × 10−9 cos ωt ) Using the formula of =
power P
V2
=
R
i.e., R
V2
P
i.e., E2 = π × 10−9 ω sin ωt For constant potential drop V1 =V2 =E

( 0.6 )=
2
And induced current in the smaller loop, E2 9
R= = Ω &,
E2 1 1
P1 0.76 19
I2= = πω × 10 −9 sin ωt A.
R 2
( 0.6 )=
2
E2
R=
2 = 0.3 Ω
Example 8: Two parallel vertical metallic rails AB and P2 1.20
CD are separated by 1 m. They are connected at the ∴ The terminal velocity of the rod is 1m/s & R1=0.47 Ω
two ends by resistances R1 and R2 as shown in Figure & R2=0.3 Ω
22.40. A horizontal metallic bar of mass 0.2 kg slides
without friction, vertically down the rails under the
Example 9: A square metal wire loop of side 10 cm and
action of gravity. There is a uniform horizontal magnetic
resistance 1 Ω is moved with a constant velocity V0 in
field of 0.6 T perpendicular to the plane of the rails. It
a uniform magnetic field of induction B=2 Wbm-2. The
2 2 . 3 0 | Electromagnetic Induction and Electromagnetic Waves

magnetic field lines are perpendicular to the plane of NS ES


the loop directed into the paper. The loop is connected For transformer the =
NP EP
to a network of resistors, each of 3 Ω. The resistances of
lead wire OS and PQ are negligible. What should be the E  4.6 × 1000 × 1000
s
speed of the loop so as to have a steady current 1 mA = N s
= N = 20,000
 Ep  p 230
in the loop? Give the direction of current in the loop.  
If Ip is current in primary, than the power in primary


coil is
V0
S PP = IP × EP = 6.9 kW
O 3 3
B 6.9 × 103
∴ Ip
= = 30 A ;
A C 230
3
P 3 Is Np 1000 1
3 &= = =
Q Ip Ns 20000 20

1 30
 Is = ×I = = 1.5 A ;
20 P 20
Sol: The network of resistors is a balanced wheatstone ∴ Current rating of the secondary coil is 1.5
bridge. The induced e.m.f. in the loop is E=BlV, where l
is one side of square loop, moving with speed v in the
magnetic field. Example 11: An infinitesimally small bar magnet of
dipole moment M is pointing and moving with the
The network mesh ASCQ is a balanced Wheatstone. So speed v in the x-direction. A small closed circular
there is no current through branch AC. conducting loop of radius ‘a’ and of negligible self-
Let R be the effective resistance of mesh ASCQ inductance lies in the y-z plane with its center at x=0,
and its axis coinciding with the x-axis. Find the force
6×6 opposing the motion of the magnet, if the resistance of
∴R = = 3Ω
6+6 the loop is R. Assume that the distance x of the magnet
Resistance of loop OSCQP =3+1=4 Ω from the center of the loop is much greater than a.

Let speed of loop through the field be V0 Sol: The flux linked with loop due to magnetic field of
bar magnet will decrease as the bar moves away from
∴ The induced E.M.F. in the loop is E=BlV0
the loop. The current induced in the loop will oppose
E = 2×0.1×V0 = 0.2V0 its cause i.e. will create a magnetic field at the location
of bar magnet such that the bar magnet is attracted
& using Ohm’s law the current in the circuit is towards the loop, thus bar magnet is decelerated.
E BlV0 0.2V0 Field due to the bar magnet at distance x (near the
I
= = =
R R 4 µ0 2M
4 × 10−3 loop) B =
−3
 I =10 A ⇒ V0 = =2 × 10−2 ms−1 4 π x3
0.2
Flux linked with the loop:
According to Fleming’s right hand rule direction of µ0 2M
φ = BA = πa2 ×
induced current in the loop is in clockwise direction. 4 π x3
e.m.f. induced in the loop:
Example 10: A power transformer is used to step up an dφ µ0 6π × Ma2 dx µ0 6πMa2
alternating e.m.f. from 230 V to 4.6kV to transmit 6.9KW E=
− = = v
dt 4 π x4 dt 4 π x 4
of power. If primary coil has 1000 turns, find
∴ Induced current:
(a) no. of turns in the secondary
E µ0 3πMa2 3µ Ma2
(b) the current rating of the secondary coil. I= = × ⋅v = 0 ⋅v
R 2π Rx 4 2Rx 4
Sol: For coil of transformer E ∝ N where E is induced (B) Find the opposing force
E.M.F. and N is number of turns in the coil.
The induced current develops field around it. As coil is
P hysi cs | 22.31

moving in the external field it will be opposed by the Example 13: A light beam travelling in the x - direction
force which is equal to heat dissipated in the coil due is described by the electric field
= Ey 300 sin ω(t − x / v)
to resistive force. . An electron is constrained to move along the
y-direction with the speed of 2.0 × 107 m/s. Find the
Heat dissipated in coil= Resistive force acting on coil
maximum electric force and the maximum magnetic
while it is in motion.
force on the electron.
I2R ;
∴ Fv = (Dimension of power)
2 Sol: The maximum force exerted by the wave is
I2R  3µ0Ma 
2 2 2 4
R 9 µ0M a v F=FE+FB= qE + qvB.
⇒ F= =   × v2 × = .
v  2Rx 4  v 4 Rx8
  (i) Maximum electric field E0 = 300 V / m

∴ Maximum electric force FE=qE0


Example 12: In an L-C circuit L=3.3 H and C=840 pF.
At t=0 charge on the capacitor is 105mC and maximum. = (1.6 × 10−19 )(300) = 4.8 × 10−17 N
Compute the following quantities at t=2.0 ms:
E
(a) The energy stored in the capacitor. (ii) From the equation, c = 0
B0
(b) The energy stored in the inductor. E
Maximum magnetic field B0 = 0
(c) The total energy in the circuit. c
300
=Or B0 = 10 −6 T
1 2 3.0 × 108
Sol: In LC circuit, the energy stored in inductor is Li
2 2 ∴ Maximum magnetic force FB= B0qv sin 90o=B0qv
and energy stored in capacitor is q .
2C
Substituting the values we have,
Given, L=3.3 H, C=840 ×10 F and qmax=105×10−6C
−12

Maximum magnetic force = (10−6 )(1.6 × 10−19 )(2.0 × 107 )


The circuit when connected to AC supply, oscillated and
= 3.2 × 10−18 N
the angular frequency of oscillations of circuit which is,
Hence total force is F= (4.8 + 0.32) × 10−17 N
1 1
ω
= = = 1.9 × 10 4 rad / s = 5.12 × 10−17 N
LC 3.3 × 840 × 10 −12

Charge stored in the capacitor at any time instant t is


given by,
= q q0 cos ωt
JEE Advanced/Boards
(a) At t= 2 × 10−3 s ; charge in capacitor is Example 1: A wire frame of area 3.92×10−4m and
resistance 20Ω is suspended from a 0.392 m long
(  )
q =105 × 10−6 cos 1.9 × 10 4  2 × 10−3 
 
thread. There is a uniform magnetic field of 0.784 T
−6 and the plane of wire-frame is perpendicular to the
=100.3 × 10 C =100 µC magnetic field. The frame is made to oscillate under
∴ Energy stored in the capacitor is gravity by displacing it through 2×10−2m from its initial
position along the direction of magnetic field. The
( )
2
1 q2 100.3 × 10 −6 plane of the frame is always along the direction of
UC
= = = 5.99 J the thread and does not rotate about it. What is the
2 C 2 × 840 × 10 −12
induced e.m.f. in a wire-frame as a function of time?
(c) Total energy in the circuit Also find the maximum current in the frame.

( )
2
−6
1 q0 105 × 10
2
Sol: As the wire frame oscillates in the magnetic field,
=U = = 6.56 J
2 C 2 × 840 × 10 −12 the angle between the area vector and the magnetic
field continuously varies. Thus, the flux linked with
(b) Energy stored in inductor in the given time the frame changes and e.m.f.and current is induced
=total energy in circuit – energy stored in capacitor in the frame. As the magnetic field is uniform, the net
magnetic force on the frame will be zero.
=6.56-6=0.56 J
The instantaneous flux through the frame when it is
displaced through an angle θ is given by
= Φ BA cos θ
2 2 . 3 2 | Electromagnetic Induction and Electromagnetic Waves

Substituting the values, we get

( 0.784 ) × (3.92 × 10−4 ) × 5 × (5 × 10−2 )


2
 =E sin10 t
 = 4 × 10−6 sin10 t
B
4 × 10−6 V and
⇒ Emax =


sin
Emax 4 × 10−6
g
mg mg cos
m
Imax = = = 2 × 10−7 A
R 20
Instantaneous induced e.m.f. to the coil is
Example 2: A variable magnetic field creates a
dΦ dθ constant e.m.f. E in a conductor ABCDA. The resistance
E=
− BA sin θ
=
dt dt of the portions ABC, CDA and AMC areR1, R2and R3,
sinceθ is very small respectively. What current will be recorded by the meter
M? The magnetic field is concentrated near the axis of

E = BA θ
dt
( sin θ = θ )  … (i) the circular conductor.

(B) Find the equation of motion & its solution


M
The force acting on the coil when it is displaced by A
small angle θ B
2 2
dx d x
m − mgsin θ or
= −gsin θ
= C
2
dt dt2
From Figure 22.43 the displacement of the coil is
x D
θ = ⇒ x = θ

d2 x d2 θ gθ
 = − gθ ⇒ =−
dt 2
dt 2 l Sol: Due to variable magnetic field, e.m.f. and current
are induced in the coil ABCDA.
Putting ω = ( g / l) , we get Let E1 and E2 be the e.m.f.s developed in ABC and CDA,
respectively. Then E1 + E2 =E.
d2 θ
+ ω2 θ = 0  … (ii) There is no net e.m.f. in the loop AMCBA as it does not
dt2
enclose the magnetic field. If E3 is the e.m.f. in AMC
This is the equation of S.H.M. then E1– E3=0. The equivalent circuit and distribution of
current is shown in Figure.
(C) Solve equation (i) to get EmaxandImax
By the loop rule R1 (x-y) + R2 x= E1+E2 =E
Solution of equation (ii) is given by θ = θ0 sin ω t
Substituting the value of θ in equation (i), we get x + -
d
(
E = BA θ0 sin ωt
dt 0
θ sin ωt) ( ) R2 R1
R3

M
= BA θ0 sin ωt ωθ0 cos ωt + +
 E2 E1
E BA ωθ20 sin2ωt
= ...(iii) ... (iii) - x-y - y
x
g  9.8  −1
Here
= ω =  =  5 rads
l  0.392  And R3 y- R1(x-y) =E3-E1 =0
x0 2 × 10−2 ER1
And θ0 = = = 5 × 10−2 rad Solving for y, y= .
l 0.392 R1R 2 + R 2R3 + R3R1
P hysi cs | 22.33

Example 3: A square loop of side ‘a’ and a straight, Sol: If n2 is the number of turns in secondary and φ2 is
infinite conductor are placed in the same plane with the flux linked through one turn, then the flux linked
two sides of the square parallel to the conductor. through the secondary is n2φ2.
The inductance and resistance are equal to L and R
Magnetic field inside any point of solenoid B = µ0 n1 i1
respectively. The frame is turned through 1800 about
where n1 is no turns in primary and i1is current in
the axis OO’. Find the electric charge that flows in the
primary.
square loop.
Flux through secondary having turns n2 is

n2 (BA ) =
O
b a n2 φ2 = µ0 n1n2i1 A
n2 φ2
⇒ M= µ0 n1n2 S
=
i1
4 π × 10−7 × 50 × 200 × 4 × 10−4
= = 5 × 10−4 H.
−2
O’ 10

di Example 5:A rectangular conducting loop in the vertical


Sol: For LR circuit, the total E.M.F. is =
E iR + L . And x-z plane has length L, width W, mass M and resistance
dt
the charge in the coil is q = I dt . ∫ R. It is dropped lengthwise from rest. At t=0 the bottom
of the loop is at a height h above the horizontal x-axis.
There is a uniform magnetic field B perpendicular to
By circuit equation iR =  ε − L di  where
dt  the x-z plane, below the x-axis. The bottom and top

of the loop cross this axis at t=t1 and t2 respectively.
di Obtain the expression for the velocity of the loop for
ε=induced e.m.f. and L = self-induced e.m.f.
dt time t1 ≤ t ≤ t2 .
Integrating above equation w.r.t time we get
Sol: The motional e.m.f. induces in the loop as it moves
di in the magnetic field. The direction of induced current
∫ Ri dt =ε
∫ dt − ∫ L dtdt will be such that the ampere force on the width of the
dφ f loop will be vertically upwards.
⇒ Rq = ∫ − dt dt − L[i]i =
φi − φf
For time t1, the loop is freely falling under gravity, so
(= ifinal 0 )
i initial 0,= velocity attained by loop at t=t1
⇒ q = ( φi − φf ) / R ν1= gt1= 2gh
Consider a strip at a distance x in the initial position. During the time t1 ≤ t ≤ t2 , flux linked with the loop is
µ I changing, so induced e.m.f.
Then B= 0 along the inward normal to the plane.
2πx

E=
− −BvW
=
µ I µ0 Ia dx dt
∴ dφi = 0 a dx cos0 =
2π x 2π x BvW
and induced current I = − clockwise
a +b
µ0 Ia dx µ0 Ia a + b R
2π b∫ x
=
⇒ φi = ln
2π b B2 vW 2
Magnetic force F=WIB = −
R
µ0 Ia 2a + b µ Ia 2a + b
Similarly φf =− ln ∴ φi − φf = 0 ln dv B2 vW 2
2π a+b 2π b So, m = mg −
dt R
µ0 Ia 2a + b
∴ q = ln mdv
2πR b dt = Integrating,
 B2 W 2 v  
 mg − 
 R  
Example 4: A straight solenoid has 50 turns per cm 
in primary and 200 turns in the secondary. The area mR  B2 ν W 2 
t=
− loge mg − +A
of cross-section of the solenoid is 4 cm2. Calculate the B2 W 2  R 
mutual inductance.
2 2 . 3 4 | Electromagnetic Induction and Electromagnetic Waves

At=t t1 , =
v v= gt1 to ampere force should be balanced by the net torque
1
of the external agent which is maintaining constant
 angular velocity of the rod.
mR B2 v 1 W 2 
A=
t1 + loge mg − 
B2 W 2 R
 
y A
Substituting for A,
x

d
 B2 vW 2 

dr

Ro
B2 W2  mg − 
− ( t −t1 ) R
= loge  

r
e mR mg FM
 B v1 W 2
2  
 mg −  O
 R 
Gives the expression for velocity of the loop in the
interval t1 ≤ t ≤ t2 . (a) As the terminals of the switch S are connected
between the points O and C, so the e.m.f. across the
switch is same as across the ends of the rod. Now to
Example 6:A metal rod OA of mass m and length l
calculate the e.m.f. across the rod, consider an element
is kept rotating with a constant angular speedwin a
of the rod of length dr at a distance r from O, then
vertical plane about a horizontal axis at the end O. The
free end A is arranged to slide without friction along dE = Bνdr = Brωdr ( as ν = rω)
a fixed conducting circular ring in the same plane as l
1
that of rotation. A uniform and constant magnetic so E =∫ Bωr dr = Bωl2 ………( i )
2
induction B is applied perpendicular and into the plane 0
And in accordance with Fleming’s right hand rule the
of rotation as shown in Figure. An inductor L and an
direction of current in the will be from A to Oand so O
external resistance R are connected through a switch S
will be at a higher potential (as inside a source of e.m.f.
between the point O and a point C on the ring to from
current flows from lower to higher potential)
an electrical circuit. Neglect the resistance of the ring
the rod. Initially, the switch is open. (b)(i) Treating the ring and rod rotating in the field as a
source of e.m.f. E given by equation (i), the equivalent
(a) What is the induced e.m.f. across the terminals of
circuit (when the switch S is closed) is as shown in
the switch?
Figure.
(b) The switch S is closed at time t=0
A
y
A S
 E
S 
O
R
C L
L C

(i) Obtain an expression for the current as a function


of time Applying Kirchhoff’s loop rule to it, keeping in mind
that current in the circuit is increasing, we get
(ii) In the steady state, obtained the time dependence
of the torque required to maintain the constant angular dI dI 1
E −=
IR − L 0 ;or= dt
speed, given that the rod OA was along the positive dt (E − IR ) L
x-axis at t=0. which on integration with initial condition I=0 at t=0
yields
Sol: As the rod rotates in uniform magnetic field,
motional e.m.f. is induced in it. When the switch is = (
I I0 1 − e− t/ τ with=)
I0
E
R
and= τ
L
R
closed, induced current flows in the coil. The direction
of current will be such that the torque on the rod due So substituting the value of E from Eqn. (i) we have
to ampere force will oppose the motion of the rod. The Bωl2 1 − e−(R /L )t 
= I … (ii)
torque, due to weight of the rod, and the torque due 2R  

P hysi cs | 22.35

Sol: As the rod moves in the magnetic field, motional


As in steady state I is independent of time, i.e., e− t/ τ →
e.m.f. is induced in it. The current in the rod will be
0 ⇒ t → ∞ ,so
such that the ampere force on it will be opposite to the
Bωl2 direction of motion. As the rod moves the resistance
Isteady state =I max = … (iii)
2R  of path increases. So to maintain constant current the
Now as the rod is rotating in a vertical plane so for the motional e.m.f. should also increase. So in turn, the
situation shown in Figure 22.48 it will experience torque velocity of the rod should increase.
in clockwise sense due to its own weight and also due Let F be the instantaneous force acting on the rod
to the magnetic force on it. So the torque on element MN at any instant t when the rod is at a distance x.
dr,=dτ (mg) × r cos θ + FM × r The instantaneous flux φ is given by φ = B × d × x
M The instantaneously induced e.m.f. is given by
i.e. dτ
=
l
( dr ) g × r cos θ + BI dr dφ  dx 
E=
− −Bd 
= 
 M  dt  dt 
× r  as = m dr andF=
M BI dr 
 l  The instantaneous total resistance of the circuit=R+2 λ x
So total torque acting on the rod Current in the circuit is

dx i (R + 2λx )
l 2
M  Mgl l E Bd  dx 
=  ∫ rdr
τ  g cos θ + BI= cos θ + BI = i (R + 2λx)  ⇒
=i dx =
l 0 2 2 ⇒R = (R + 2λx )  dt  dt Bd
dt Bd i (R + 2λx )
But as rod is rotating at constant angular velocity ω, i (R + 2λx )i.e., velocity =
Bd
θ =ωt and from equation (iii) =
I (
Bωl2 / 2R ) i.e., velocity =
Bd
2 4
Mgl B ωl The instantaneous acceleration
So, τ
= cos ωt + … (iv)
2 4R 
d2 x 2iλ  dx 
And hence the rod will rotate at constant angular = a =  
velocity ω if a torque having magnitude equal to that dt2 Bd  dt 
given by equation is applied to it in anticlockwise
=
( 2
)
2iλ  i R + 2λx  2i λ R + 2λx
=  
( )
sense.
Bd  Bd 2 2
 Bd

M
∴ Instantaneous applied force
2i2 λ (R + 2λx )
F ma
= = ×m
B d B2d2
R
FB2d2
From this equation i2 =
x 2mλ (R + 2λx )

FB2d2
N Heat produced per second = i2 (R + 2λx ) =
2mλ
i (R + 2λx )
Example 7: Two long parallel horizontal rails at Power W= F. v= F ×
Bd
distance d apart and each having a resistance λ per
2 2
HeatR.product
unit length, are joined at one end by a resistance A H FBHeatd productBd H FB2d2 Bd
Therefore,
= = × = = ×
perfectly conducting rod MN of mass m is free to work
slidedone W 2mwork (
λ FidoneR + 2 λ x )W 2m λ Fi ( + 2λx )
R
on rails without friction. There is a uniform magnetic
B3d3 B3d3
field of induction B normal to the plane of the paper = =
2mλ (R + 2λx ) 2mλ (R + 2λx )
and direct into the paper. A variable force F is applied
to the rod MN such that, as the rod moves, a constant
current flows through R.
Example 8: A metal rod of mass m can rotate about
(a) Find the velocity of the rod and the applied force F a horizontal axis O, sliding along a circular conductor
as function of the distance x of the rod from R. of radius a. The arrangement is located in a horizontal
(b) What fraction of the work done per second by F is and uniform magnetic field of induction B directed
converted into heat? perpendicular to the ring plane. The axis and the ring
2 2 . 3 6 | Electromagnetic Induction and Electromagnetic Waves

are connected to an e.m.f. source to form a circuit of between the rails d is large compared with x, the distance
resistance R. Deduce the relation according to which between the wires. Both wires and rails are made of a
the source e.m.f. must vary to make the rod rotate material of resistivity ρ per unit length. A magnetic flux
with a constant angular velocityω. Neglect the friction, of density B applied perpendicular to the rectangle
circuit inductance and ring resistance. made by the wires and rails. One wire is moved along
the rails with a uniform speed v while the other is held
E
stationary. Determine how the force on the stationary
wire varies with x and show that it vanishes for a value
µ0 v
of x approximately equal to .
t 4 πρ

x Rail 1
O mg
x dx
B
I v
d
Sol: As current flows in the rod due to the source e.m.f., B
it experiences torque due to ampere forces and starts Rail 2
rotating. The torque due to weight of the rod should
balance the torque due to ampere force to maintain
Sol: Due to motional e.m.f. current will be induced in
constant angular velocity. As torque due to weight of
rectangular loop. The stationary wire will be attracted
the rod varies with angular position the torque due to
by the moving wire, as well as it will experience a force
ampere force should also vary. So in turn, the current
due to the uniform magnetic field.
and thus source e.m.f. should also vary.
Let at any instant t, during the motion of second wire,
Inductance e.m.f. across the ends of the rod
a the second wire is at a distance x. The area of the
1
E =∫ dE =∫ Bωx dx = B ωa2 rectangle between the two wires is xd. Rate of change
0
2 of magnetic flux through the rectangle
Force on the rod if a current I flow through it:
dφ d dx
F= IaB =
dt dt
(=
B x d) B=
d
dt
Bvd

If the angular velocity is constant so that torque about


∴ Induced e.m.f.
O must vanish. Hence

a 1 dφ
mg sin ωt = I a2 B e=
− − Bvd
=
2 2 dt
So, the current induced in the rectangle I is given by
∴ Current required through the rod
E Bvd
mgsin ωt I= = −
I= R 2 (d + x ) ρ
aB
The force between the two wires due to current flow
This must be equal be to the current due to total e.m.f.
in the circuit µ0i1i2 µ0 2I2d
F
= .d
= ×
1 2πx 4π x
E − Bωa2 2
2 mg sin ωt
=I = ; µ  2d   Bvd 
R aB = 0   
4π  x   2 ( d + x ) ρ 
 
=∴E
1
2Ba
(
2 mgR sin ωt + B2 ωa2 ) The force F’, due to magnetic field on the stationary
wires
Example 9: Two long wires are placed on a pair of  Bvd  B2d2 v
parallel rails perpendicular to the wires. The spacing F' B=
= I d Bd  = 
 2 ( d + x ) ρ  2 ( d + x ) ρ
P hysi cs | 22.37

The former force on stationary wire will be directed


dP Li2  1 2 
towards left hand side because opposite currents repel = 0 ⇒ 0  − e− t/ τ + e−2t/ τ  =
0
dt τ  τ τ 
each other while the force due to magnetic field will be
directed toward right hand according to Fleming’s left 1
⇒ − e− t/ τ =
hand rule. 2

∴ Fresultant =
F'− F Putting in (i)
2 Li20  1 1  LE2 E2
B2d2 v µ  2d   Bdv  Pmax
=  − =  =
= − 0    τ  2 4  4R (L / R ) 4R
2
2 ( d + x ) ρ 4 π  x   2 ( d + x ) ρ 

B2d2 v  µ0 dv 
Example 11:A parallel-plate capacitor having plate area
= 1 − 
2 ( d + x ) ρ  4 π ( d + x ) ρ  A and plate separation d is joined to a battery of emf
V and internal resistance 2R, at t=0. Consider a plane
The force will be zero, when surface of area A, parallel to the plates and situated
µ0 dv µ0 v symmetrically between them. Find the displacement
= 1 or x = current through this surface as a function of time. [The
4π ( d + x ) ρ 4 πρ
charge on the capacitor at time t is given by q=CV
(Neglecting x in comparison with d). (1 − e− t / τ ) .where τ =CR]
d ΦE
Sol: id = ε0 is the displacement current, ϕEis
Example 10: An inductance L and a resistance R are dt
connected in series with a battery of e.m.f. . Find the the flux of the electric field between the plates of the
maximum rate at which the energy is stored in the capacitor.
magnetic field.
Given, q=CV (1 − e− t / τ )
Sol: Substitute the expression for instantaneous current q CV
∴ Surface charge density σ= = (1 − e−t/ τ )
in the LR series circuit in the formula for energy stored A A
in the inductor. Electric field between the plates of capacitor,
I σ CV
E
= = (1 − e− t / τ )
ε0 ε0 A

R +O Electric flux from the given area,


-C E
CV
L φE= EA= (1 − e− t / τ )
ε0
dφE
Displacement current, id = ε0
dt
The energy in the magnetic field at time t is, d  CV  CV −t/ τ
ε0  (1 − e−t/ τ ) =
Or, id = e
1 2 1 2
( )
2
=U = Li Li 1 − e−t/ τ dt  ε0  τ
2 2 0
The rate at which the energy is stored is Substituting,
= τ CR
= ′ where R ′ 2R

V − t/2CR
dU
P= =
dt
Li20 1 − e− t/ τ( )( −e )  − 1τ 
− t/ τ We have, id =
2R
e

Li20 ε0 A
=
τ
(e − t/ τ
)
− e−2t/ τ ... ( i ) Again substituting, C =
d
td
This rate will be maximum when V 2ε0 AR
id = e
2R
2 2 . 3 8 | Electromagnetic Induction and Electromagnetic Waves

JEE Main/Boards

Exercise 1 Q.9 A square copper coil of each side 8 cm consists of


100 turns. The coil is initially in vertically plane, such
Q.1 Can a person sitting in a moving train measure the that the plane of coil is normal to the uniform magnetic
potential difference between the ends of the axle by a field of induction 0.4 weber m-2. The coil is turned
sensitive voltmeter? through 1800 about a horizontal axis in 0.2s. Find the
induced e.m.f.

Q.2 A coil of mean area 500 cm2 and having 1000 turns
is held perpendicular to a uniform field of 4 × 10−4 T. Q.10 A 5 H inductor carries a steady current of 2 A. How
can a 50 V self-induced e.m.f. be made to appear in the
1
The coil is turned through 1800 in s. Calculate the inductor?
average induced e.m.f.. 10

Q.11 A conducting wire of 100 turns is wound over 1


Q.3 The self-inductance of an inductance coil having cm near the center of a solenoid of 100 dm length and
100 turns is 20 mH. Calculate the magnetic flux through 2 cm radius having 1000 turns. Calculate coefficient of
the cross-section of the coil corresponding to a current mutual inductance of the two solenoids.
of 4 mA. Also find the total flux.
Q.12 If the self-inductance of an air core inductor
Q.4 A rectangular loop of wire is being withdrawn out increases from 0.01 mH to 10 mH on introducing an
of the magnetic field with velocity v. The magnetic field iron core to it, what is the relative permeability of the
is perpendicular to the plane of paper. What will be the core used?
direction of induced current, in the loop?
Q.13 State Lenz’s law. The P Q
Q.5 A solenoidal coil has 50 turns per centimeter along closed loop PQRS is
its length and cross sectional area of 4 × 10−4 m2. 200 moving into uniform
turns of another wire is wound round the first solenoid magnetic field acting at
coaxially. The two coils are electrically insulted from right angle to the plane of
each other. Calculate the mutual inductance between the paper as shown in the
the two coils. Figure. State the direction
S R
in which the induced
Q.6 Calculate the mutual inductance between two coils, current flows in the loop.
when a current of 4.0 A changes to 8.0 A in 0.5 second
and induces an e.m.f. of 50 m V in the secondary coil. Q.14 A solenoid with an iron core and a bulb are
connected to a D.C. source. How does the brightness of
Q.7 In a car spark coil, an e.m.f. of 40,000 V is induced the bulb change, when the iron core is removed from
in the secondary coil when the primary coil current the solenoid?
changes from 4 A to 0 A in 10 µ s. Calculate the mutual
inductance between the primary secondary windings Q.15 What is induced e.m.f.? Write faraday’s law of
of this spark coil. electromagnetic induction. Express it mathematically.
A conducting rod of length ‘l’, with one end pivoted,
Q.8 A current of 10 A is flowing in a long straight wire is rotated with a uniform angular speed ' ω' in a vertical
situated near a rectangular coil. The two sides, of the plane, normal to a uniform magnetic field ‘B’. Deduce
coil, of length 0.2 m are parallel to the wire. One of an expression for the e.m.f. induced in this rod.
them is at a distance of 0.05m and the other is at a
distance of 0.10 m from the wire. The wire is in the plane
Q.16 A circular coil of radius 8 cm and 20 turns rotates
of the coil. Calculate the magnetic flux through the
about its vertical diameter with an angular speed of 50 s-1
rectangular coil. If the current decays uniformly to zero
in a uniform horizontal magnetic field of magnitude
in 0.02s, find the e.m.f. induced in the coil and indicate
3 × 10 −2 T. Find the maximum and average value of the
the direction in which the induced current flows.
e.m.f. induced in the coil.
P hysi cs | 22.39

Q.17 Define self-inductance and give its S.I. unit. Derive P R


an expression for self-inductance of a long, air-cored
solenoid of length l, radius r, and having N number of
 v
turns.

S
Q.18 A 0.5 m long metal rod PQ completes the
circuit as shown in the Figure. The area of the circuit Deduce an expression for
is perpendicular to the magnitude field of flux density
0.15 T. If the resistance of the total circuit is 3 Ω, calculate (i) The e.m.f. induced across the arm ‘RS’,
the force needed to move the rode in the direction as (ii) The external force required to move the arm, and
indicated with a constant speed of 2 ms-1.
(iii) The power dissipated as heat.
Q
Q.23 Define self-inductance of a coil. Write its S.I. units.

P Q 24 The identical loops, one of copper and the other


of aluminum, are rotated with the same angular speed
Q.19 What are eddy currents? How are these produced? in the same magnetic field. Compare (i) the induced
In what sense are eddy currents considered undesirable e.m.f. and (ii) the current produced in the two coils.
in a transformer and how are these reduced in such a Justify your answer.
device?
Q.25 Two bar magnets are quickly moved towards a
Q.20 The circuit arrangement given below shows that metallic loop connected across a capacitor ‘C’ as shown
when an a.c. passes through the coil A, the current in the Figure. Predict the polarity of the capacitor.
starts flowing in the coil B.
C

S N S N

Q.26 A bar magnetic is moved in the direction indicated


by the arrow between two coils PQ and CD. Predict the
directions of induced current in each coil.
Coil A Coil B
P Q C D
(i) State the underlying principle involved. N S
 
(ii) Mention two factors on which the current produced A A
in the coil B depends.

Q21 (i) State faraday’s law of electromagnetic induction.


Exercise 2
(ii) A jet plane is travelling towards west at a speed of Single Correct Choice Type
1800 km/h. what is the voltage difference developed
between the ends of the wing having a span of 25m, Q.1. A conducting loop of radius R is present in a
if the earth’s magnetic field at the location has a uniform magnetic field B perpendicular to the plane of
magnitude of 5 × 10 −4 T and the dip angle is 300? the ring. If radius R varies as a function of time‘t’, as
R=R0+t. The e.m.f. induced in the loop is
Q.22 (a) Write the two applications of eddy currents. (b)
Figure 22.57 shows a rectangular conducting lop PQSR
in which arm RS of length '  ' is movable. The loop is
kept in a uniform magnetic field ‘B’ directed downward R
B
perpendicular to the plane of the loop. The arm RS is
moved with a uniform speed ‘v’.
2 2 . 4 0 | Electromagnetic Induction and Electromagnetic Waves

(A) 2π(R0+t)B clockwise (B) π(R0+t)B clockwise (C) The current in each will remain the same
(C) 2π(R0+t)B anticlockwise (D) zero (D) The current in one will increase and in other will
decrease
Q.2 A square wire loop of 10.0 cm side lies at right
angle to a uniform magnetic field of 20T. A 10V light Q.6 The Figure shows an
bulb is in a series with the loop as shown in the Figure. isosceles triangle wire frame with B
The magnetic field is decreasing steadily to zero over apex angle equal to π / 2 . The o
frame starts entering into the 90
a time interval ∆t . The bulb will shine full brightness if v
∆t is equal to uniform magnetic field B with
Constant velocity v at t=0. The
t=0
longest side of the frame is
perpendicular to the direction of velocity. If i is the
instantaneous current through the frame then choose
the alternative showing the correct variation of i with
time.

i i
(A) (B)
(A) 20 ms (B) 0.02 ms t t
(C) 2 ms (D) 0.2 ms
i i
(C) (D)
Q.3 The dimensions of permeability of free space can
t t
be given by

(A) MLT −2 A −2  (B) MLA −2  Q.7 A thin wire of length 2 m is perpendicular to the xy
    
( )
plane. It is moved with velocity v = 2iˆ + 3ˆj + kˆ m / s
(C) ML−3 T2 A2 
 
(D) MLA −1 
  through a region of magnetic induction B= (ˆi + 2ˆj)
Wb/ m2. Then potential difference induced between the
Q.4 A vertical magnet is dropped from position on the ends of the wire:
axis of a fixed metallic coil as shown in Figure, figure (i).
(A) 2 V (B) 4 V
In figure (ii) the magnet is fixed and horizontal coil is
dropped. The acceleration of the magnet and coil are (C) 0 V (D) none of these
a1 and a2 respectively then
Q.8 A long metal bar of 30 cm length is aligned along a
S north south line and moves eastward at a speed of 10
ms-1. A uniform magnetic field of 4.0 T points vertically
S
downwards. If the south end of the bar has a potential
N of 0 V, the induced potential at the end of the bar is
Fixed Fixed N (A) +12 V

(i) (ii) (B) -12 V


(C) 0 V
(A) a1 > g,a2 > g (B) a1 > g,a2 < g
(D) Cannot be determined since there is not closed
(C) a1 < g,a2 < g (D) a1 < g,a2 > g
circuit

Q.5 Two identical coaxial circular loops carry a current


Q.9 A conducting rod moves with constant velocity
I each circulating in the same direction. If the loops
v perpendicular to the long, straight wire carrying a
approach each other
current I as shown compute that the e.m.f. generated
(A) The current in each will decrease between the ends of the rod.
(B) The current in each will increase
P hysi cs | 22.41

Q.13 When a ’J’ shaped conducting rod is rotating in


r its own plane with constant angular velocity ω , about
I one of its end P, in a uniform magnetic field B (directed
normally into the plane of paper) then magnitude of
l 
e.m.f. induced across it will be

l L
µ0 ν Il µ0 ν Il
(A) (B) Q
πr 2πr
2µ0 ν Il µ0 ν Il
(C) (D) P 
πr 4 πr

Q.10There is a uniform field B normal to the xy plane. A 1


2 2
(A) Bω L + l (B) BωL2
conductor ABC has length AB=l1, parallel to the x-axis, 2
and length BC=l2, parallel to the y-axis. ABC moves
(C)
1
2
( )
Bω L2 + l2
1
(D) Bω l2
2
in the xy plane with velocity v i + v j . The potential x y
difference between A and C is proportional to
Q.14 A metal disc rotates freely, between the poles of
(A) VXl1 + Vy l2 (B) VXl2 + Vy l1 a magnet in the direction indicated. Brushes P and Q
make contact with the edge of the disc and the metal
(C) VXl2 − Vy l1 (D) VXl1 − Vy l2 axle. What current, if any, flows through R?

N
Q.11 A conducting rod PQ of length 5 m oriented as
shown in Figure is moving with velocity 2 î m/s without

( )
R
any rotation in a uniform magnetic field 3ˆj + 4kˆ T. e.m.f. P 
Q
induced in the rod is
y S
Q
2m/s
5m (A) A current from P to Q

53
o (B) A current from Q to P
x (C) No current, because the e.m.f. induced in one side
P
of the disc is opposed by the back e.m.f.
(A) 32 V (B) 40 V (c) 50 V (D) none (D) No current, because the e.m.f. induced in one side
of disc is opposed by the e.m.f. induced in the other
Q.12 The magnetic field in a region is given by side
x ˆ (E) No current, because no radial e.m.f. is induced in
B B0 1 +
= k . A square loop of edge length d is
a the disc
placed with its edge along x & y axis. The loop is moved

with constant velocity V = V0 ˆi . The e.m.f. induced in the Q.15 A rectangular coil of single turn, having area A,
loop is rotates in a uniform magnetic field B with an angular
velocity w about an axis perpendicular to the field. If
V0 B0 d2 V0 B0 d2 initially the plane of coil is perpendicular to the field,
(A) (B)
a 2a then the average induced e.m.f. when it has rotated
V0 B0 a2 through 900 is
(C) (D) none
d
2 2 . 4 2 | Electromagnetic Induction and Electromagnetic Waves

j
C L
A B
A
B

i
(A) Remains stationary
ωBA ωBA ωBA 2ωBA
(A) (B) (C) (D)
π 2π 4π π (B) Is attracted by the loop-A
(C) Is repelled by the loop-A
Q.16 A copper rod AB of length L, pivoted at one end
(D) Rotates about its CM, with CM fixed
A, rotates at constant angular velocity ω , at right angle
to a uniform magnetic field of induction B. The e.m.f.
developed between the midpoint C to of the rod and Q.21 A circular loop of radius R, carrying current I, lies
end B is in x-y plane with its center at origin. The total magnetic
flux through x-y plane is
Bωl2 Bωl2 3Bω l2 3Bω l2
(A) (B) (C) (D)
4 2 4 8 (A) Directly proportional to I
(B) Directly proportional to R
Q. 17 Figure 22.70shows a uniform magnetic field
(C)Directly proportional to R2
B confined to a cylindrical volume and is increasing
at a constant rate. The instantaneous acceleration (D) Zero
experienced by an electron placed at P is
Q.22 Two circular coils can be arranged in any of
the three situations in the Figure 22.72. Their mutual
P inductance will be

(a) (b) (c)


(A) Zero (B) Towards right
(C) Towards left (D) Upwards (A) Maximum in situation (a)
(B) Maximum in situation (b)
Q.18 A small coil of radius r is placed at the center
(C) Maximum in situation (c)
of a large coil of radius R, where R>>r. The coils are
coplanar. The coefficient of mutual inductance between (D) The same in all situations
the coils is
µ0 πr µ0 πr 2 µ0 πr 2 µ0 πr
(A) (B) (C) 2
(D) Previous Years’ Questions
2R 2R 2R 2R 2
Q.1 A conducting square loop of side L and resistance
Q.19 A long straight wire is placed along the axis of R moves in its plane with a uniform velocity ν
circular ring of radius R. The mutual inductance of this perpendicular to one of its sides. A magnetic induction
system is B, constant in time and space, pointing perpendicular
µ0R µ0 πR µ0 to and into the plane of the loop exists everywhere. The
(A) (B) (C) (D) 0
2 2 2 current induced in the loop is  (1989)

Q.20 Two identical circular loops of metal wire are lying


on a table without touching each other. Loop-A carries v
a current which increases with time. In response, the
loop-B
P hysi cs | 22.43

(A) BLv/R clockwise (B) BLv/R anticlockwise the circular region (2000)
(C) 2BLv/R anticlockwise (D) Zero B(t)
P
Q.2 A thin semicircular conducting ring of radius R is r
falling with
 its plane vertical in a horizontal magnetic
induction B. At the position MNQ the speed of the ring
is ν and the potential difference developed across the
ring is (1996) a


B
(A) is zero (B) decreases as 1/r
N (C) increases as r (D) decreases as 1/r2

Q.7 A coil of wire having finite inductance and


M Q
resistance has a conducting ring placed co-axially
(A) Zero within it. The coil is connected to a battery at time t=0,
so that a time dependent current I1 (t) starts flowing
(B) BνπR 2 / 2 and M is at higher potential
through the coil. I2 (t) is the current induced in the
(C) πBRν and Q is at higher potential ring and B (t) is the magnetic field at the axis of the
coil due to I1 (t) then as a function of time (t>0), the
(D) 2RBν and Q is at higher potential
product I2 (t) B(t) (2000)

Q.3 A metal rod moves at a constant velocity in a direction (A) Increases with time
perpendicular to its length. A constant magnetic field (B) Decreases with time
exist in space in a direction perpendicular to the rod as
(C) Does not vary with time
well as its velocity. Select the correct statement (s) from
the following.  (1998) (D) Passes through a maximum
(A) The entire rod is at the same electric potential
Q.8 A metallic square loop ABCD is moving in its
(B) There is an electric field in the rod
own plane with velocity ν in a uniform magnetic field
(C) The electric potential is higher at the center of the perpendicular to its plane as shown in the Figure 22.84.
rod and decrease towards its ends Electrical field is induced  (2001)
(D) The electric potential is lowest at the center of the
rod and increase towards its ends

Q.4 A small square loop of wire of side l is placed A B


inside a large square of wire of side L (L>>l). The loops
are coplanar and their centers coincide. The mutual
inductance of the system is proportional to  (1998) v

(A) l / L (B) l2 / L (C) L / l (D) L2 / l

Q.5 A coil of inductance 8.4 mH and resistance 6 Ω is


connected to a 12 Ω battery. The current in the coil is
1A at approximately the time (1999)
(A) 500s (B) 20s (C) 35 ms (D) 1 ms (A) In AD, but not in BC
(B) In BC, but not in AD
Q.6 A uniform but time-varying magnetic field B (t)
(C) Neither in AD nor in BC
exists in a circular region a and is directed into the plane
of the paper as shown. The magnitude of the induced (D) In both AD and BC
electric field at point P at a distance r from the center of
2 2 . 4 4 | Electromagnetic Induction and Electromagnetic Waves

Q.9 As shown in the Figure, P and Q are two coaxial Q.13 In an LCR circuit as shown below both switches
conducting loops separate by some distance. When are open initially. Now switch S1 is closed, S2 kept open.
the switch S is closed, a clockwise current IP flows in (q is charge on the capacitor and τ =RC is capacitive
P (as seen by E) and an induced current IQ1 flows in Q. time constant). Which of the following statement is
The switch remains closed for a long time. When S is correct? (2013)
opened, a current IQ2 flows in Q. Then the direction IQ1 V
and IQ2 (as seen by E) are (2002)
P
Q R
S1
E
C
S
S2
L
Battery (A) At t=τ, q=CV / 2
(B) At t=2τ, q=CV(1−e−2)
(A) Respectively clockwise and anticlockwise
τ
(B) Both clockwise Attt= =
(C) = , q=CV
q CV(1−e e−) 1 )
(1 − −1
2
(C) Both anticlockwise
(D) Work done by the battery is half of the energy
(D) Respectively anticlockwise and clockwise
dissipated in the resistor.

Q.10 A short-circuited coil is placed in a time varying


Q.14 A circular loop of radius 0.3 cm lies parallel to a
magnetic field. Electric power is dissipated due to the
much bigger circular loop of radius 20 cm. The centre
current induced in the coil. If the number of turns were
of the small loop is on the axis of the bigger loop. The
to be quadrupled (four time) and the wire radius halved,
distance between their centres is 15 cm. If a current
the electrical power dissipated would be (2002)
of 2.0 A flows through the smaller loop, then the flux
(A) Halved (B) The same linked with bigger loop is (2013)
(C) Doubled (D) Quadrupled
(A) 6 × 10−11 weber (B) 3.3 × 10−11 weber
(C) 6.6 × 10−9 weber (D) 9.1 × 10−11 weber
Q.11 An electromagnetic wave in vacuum has the
 
electric and magnetic fields E and B , which are
Q.15 The magnetic field in a travelling electromagnetic
always perpendicular to each other. The direction of
 wave has a peak value of 20 nT. The peak value of
polarization is given by X and that of wave propagation electric field strength is : (2013)

by k . Then : (2012)
          (A) 6 V/m (B) 9 V/m (C) 12 V/m (D) 3 V/m
(A) X ||B and k || B × E (B) X ||E and k || E × B
         
(C) X ||B and k || E × B (D) X ||E and k || B × E Q.16 Match List-I (Electromagnetic wave type) with List-
II (Its association / application) and select the correct
Q.12 A coil is suspended in a uniform magnetic field, option from the choices given below the lists: (2014)
with the plane of the coil parallel to the magnetic lines
of force. When a current is passed through the coil it
List - I List - II
starts oscillating; it is very difficult to stop. But if an
aluminium plate is placed near to the coil, it stops. This (a) Infrared (i) To treat muscular strain
is due to : (2012) waves
(A) development of air current when the plate is placed. (b) Radio waves (ii) For broadcasting
(B) induction of electrical charge on the plate (c) X-rays (iii) To detect fracture of
bones
(C) shielding of magnetic lines of force as aluminium is
a paramagnetic material. (d) Ultraviolet (iv) Absorbed by the ozone
rays layer of the atmosphere
(D) electromagnetic induction in the aluminium plate
giving rise to electromagnetic damping.
P hysi cs | 22.45

(A) (a) → (iii), (b) → (ii), (c) → (i), (d) → (iv) R L

(B) (a) → (i), (b) → (ii), (c) → (iii), (d) → (iv)


(C) (a) → (iv), (b) → (iii), (c) → (ii), (d) → (i)
C
(D) (a) → (i), (b) → (ii), (c) → (iv), (d) → (iii)

Q.17 During the propagation of electromagnetic waves


in a medium: (2014)
2 2
Q Max Q Max

L2 L2
(A) Electric energy density is equal to the magnetic (A) (B)
L1 L1
energy density.
t t
(B) Both electric and magnetic energy densities are
zero.
(C) Electric energy density is double of the magnetic
2 2
Q Max QMax
energy density. (C) (D) L1
Q0 (For both L1 and L0)
(D) Electric energy density is half of the magnetic L2
energy density. t t

Q.18 In the circuit shown here, the point ‘C’ is kept Q.21 Two long current carrying
connected to point ‘A’ till the current flowing through thin wires, both with current I, are L
the circuit becomes constant. Afterward, suddenly, held by insulating threads of 
point ‘C’ is disconnected from point ‘A’ and connected length L and are in equilibrium as
to point ‘B’ at time t=0. Ratio of the voltage across shown in the figure, with threads I I
resistance and the inductor at t=L/R will be equal to : making an angle ' θ ' with the
 (2014) vertical. If wires have mass λ per unit length then the
A R value of I is: (g=gravitational acceleration) (2015)
C
B πλgL πgL
L (A) 2sin θ (B) 2 tan θ
µ0 cos θ µ0

πλgL πλgL
1−e e (C) tan θ (D) sin θ
(A) -1 (B) (C) (D) 1 µ0 µ0 cos θ
e 1−e

Q.19 An inductor (L=0.03 H) and a resistor (R=0.15 kΩ) Q.22 Two identical wires A and B, each of length '  ' ,
are connected in series to a battery of 15 V EMF in a carry the same current I. Wire A is bent into a circle of
circuit shown. The key K1 has been kept closed for a radius R and wire B is bent to form a square of side ‘a’. If
long time. Then at t=0, K1 is opened and key K2 is closed BA and BB are the values of magnetic field at the centres
simultaneously. At t=1 ms, the current in the circuit will BA
of the circle and square respectively, then the ratio
be (e5 ≅ 150)  (2015) BB
(A) 67 mA (B) 6.7 mA is:  (2016)
π2 π2 2 π2
(C) 0.67 mA (D) 100 mA (A) (B) (C) π (D)
16 2 16 8 2 8

Q.20 An LCR circuit is equivalent to a damped pendulum.


Q.23 Arrange the following electromagnetic radiations
In an LCR circuit the capacitor is charged to Q0 and then
per quantum in the order of increasing energy :(2016)
connected to the L and R as shown. If a student plots
2
graphs of the square of maximum charge (QMax ) on the (1) : Blue light (2) : Yellow light
capacitor with time (t) for two different values L1 and L2
(3) : X-ray (4) : Radiowave
(L1 > L2) of L then which of the following represents this
graph correctly? (Plots are schematic and not drawn to (A) (1), (2), (4), (3) (B) (3), (1), (2), (4)
scale) (2015) (C) (2), (1), (4), (3) (D) (4), (2), (1), (3)
2 2 . 4 6 | Electromagnetic Induction and Electromagnetic Waves

Q.24 Hysteresis loops for two magnetic materials A and These materials are used to make magnets for electric
B are given below : generators, transformer core and electromagnet core.
B B Then it is proper to use: (2016)
(A) A for electromagnets and B for electric generators
(B) A for transformers and B for electric generators
H (C) B for electromagnets and transformers
(D) A for electric generators and transformers

(A) (B)

JEE Advanced/Boards

Exercise 1 Q.4 A rectangular loop with a sliding connector of


length l=1.0 m is situated in a uniform magnetic field
B=2T perpendicular to the plane of loop. Resistance of
Q.1 The horizontal component of the earth’s magnetic
connector is r= 2 Ω . Two resistances of 6 Ω and 3 Ω are
field at a place is 3 × 10 −4 T and the dip is tan-1 (4/3). A
connected as shown in Figure. Find the external force
metal rod of length 0.25 m placed in the north- south
required to keep the connector moving with a constant
position is moved at a constant speed of 10 cm/s
velocity V=2m/s.
towards the east. Find the e.m.f. induced in the rod. 
+B
Q.2 A wire forming one cycle sine curve is moved in x-y 6 3

V Vx ˆi + Vy ˆj.  There exist a magnetic
plane with velocity =

field is B = − B0 kˆ . Find the motional e.m.f. develop
Q.5 Two concentric and coplanar circular coils have
across the ends PQ of wire.
radii a and b(>>a) as shown in Figure. Resistance of
y the inner coil is R. Current in the outer coil is increased
 from 0 to i, then find the total charge circulating the
B
inner coil.
P O
x
b
a

Q.3 A conducting circular loop is placed in a uniform


Q.6 A horizontal wire is free to slide on the vertical rails
magnetic field of 0.02 T, with its plane perpendicular to
of a conducting frame as shown in Figure. The wire has
the field. If the radius of the loop starts shrinking at a
a mass m and length l and the resistance of the circuit is
constant rate of 1.0 mm/s, then find the e.m.f. induced
R. If a uniform magnetic field B is directed perpendicular
in the loop, at the instant when the radius is 4 cm.
to the frame, then find the terminal speed of the wire as
it falls under the force of gravity.
C l

m

B
A B
R
P hysi cs | 22.47

Q.7 A metal rod of resistance 20Ω is fixed along a with the rails starts at vertex at the time t=0 & moves
diameter of a conducting ring of radius 0.1 m and lies symmetrically with a constant velocity of 5.2 m/s to the
on x-y plane. There is a magnetic field B= (50T) K̂ . The right as shown in Figure. A 0.35 T magnetic field points
ring rotates with an angular velocity ω=20 rad/s about out of the page. Calculate:
its axis. An external resistance of 10 Ω is connected
across the center of the ring and rim. Find the current B
through external resistance.
5.2m/s
o
90
Q.8 A triangular wire frame (each side=2m) is placed in
a region of time variant magnetic field
Having dB/dt= 3 T/s. The magnetic field is
perpendicular to the plane of the triangle. The base of
the triangle AB has a resistance 1 Ω while the other (i) The flux through the triangle by the rails & bar at
two sides have resistance 2 Ω each. The magnitude of t=3.0s
potential difference between the points A and B will be.
(ii) The e.m.f. around the triangle at that time.

Q.9 A uniform magnetic field of 0.08 T is directed into (iii) In what manner does the e.m.f. around the triangle
the plane of the page and perpendicular to it as shown vary with time?
in the Figure. A wire loop in the plane of the page has
constant area 0.010m2. The magnitude of magnetic Q.14A wire is bent into 3 circular segments of radius
field decrease at a constant rate 3 × 10 −4 Ts-1. Find the r=10cm as shown in Figure. Each segment is a quadrant
magnitude and direction of the induced e.m.f. in the of a circle, ab lying in the xy plane, bclying in the yz
loop. plane & ca lying in the zx plane.
z

c
B

r r
b
Q.10 There exists a uniform cylindrically symmetric y
magnetic field directed along the axis of a cylinder but
varying with time as B=kt. If an electron is released from
rest in this filed at a distance ‘r’ from the axis of cylinder, a
its acceleration, just after it is released would be (e and x
m are the electronic charge and mass respectively) (i) If a magnetic field B points in the positive x direction,
what is the magnitude of the e.m.f. developed in the
Q.11 A uniform but time varying magnetic field B=Kt–C; wire, when B increases at the rate of 3 mT/s?
(0 ≤ t ≤ C/K), where K and C are constants and t is time, (ii) What is the direction of the current in the segment
is applied perpendicular to the plane of the circular bc.
loop of radius ’a’ and resistance R. Find the total charge
that will pass around the loop.
Q.15 Consider the possibility of a new design for an
electric train. The engine is driven by the force due to
Q.12 A charged ring of mass m=50gm, charge 2 the vertical component of the earth’s magnetic field on
coulomb and radius R=2m is placed on a smooth a conducting axle. Current is passed down one coil, into
horizontal surface. A magnetic field varying with at a a conducting wheel through the axle, through another
rate of (0.2t) T/ s is applied on to the ring in a direction conducting wheel & then back to the source via the
normal to the surface of ring. Find the angular speed other rail.
attained in a time t1=10 s.
(i) What current is needed to provide a modest 10-KN
force? Take the vertical component of the earth’s field
Q.13 Two straight conducting rails form a right angle
be 10 µ T & the length of axle to be 3.0 m.
where their ends are joined. A conducting bar contact
2 2 . 4 8 | Electromagnetic Induction and Electromagnetic Waves

(ii) How much power would be lost for each Ω of Q.19 A rectangular frame ABCD made of a uniform
resistivity in the rails? metal wire has a straight connection between E & F
made of the same wire as shown in the figure. AEFD is
(iii) Is such a train realistic?
a square of side 1m & EB= FC=0.5 m. The entire circuit
is placed in a steadily increasing uniform magnetic field
Q.16 A rectangular loop of dimensions l& w and directed into the place of the paper & normal to it. The
resistance R moves with constant velocity V to the right rate change of the magnetic field is 1T/s, the resistance
as shown in the Figure. It continues to move with same per unit length of the wire is 1 Ω/m. Find the current in
speed through a region containing a uniform magnetic segments AE, BE &EF.
field B directed into the plane of the paper &extending
A E B
a distance 3 W. sketch the flux, induced e.m.f. & external
force acting on the as a function of the distance.

3w B

v B D F C

S Q.20 A pair o parallel horizontal conducting rails of


negligible resistance shorted at one end is fixed on a
table. The distance between the rails is L. A conducting
w massless rod of resistance R can slide on the rails
frictionally. The rod is tied to a massless string which
passes over a pulley fixed to the edge of the table. A mass
Q.17 A long straight wire is arranged along the m, tied to the other end of the string hangs vertically.
symmetry a toroidal coil of rectangular cross-section, A constant magnetic field B exists perpendicular to the
whose dimensions are gives in the Figure. The number table. If the system is released from rest, calculate:
of turns on the coil is N, and relative permeability of the
surrounding medium is unity. Find the amplitude of the
e.m.f. induced in this coil, if the current i=im cos ω t flows
along the straight wire.
b

h
m

(i) The terminal velocity achieved by the rod.


(ii) The acceleration of the mass at the instant when the
Q.18 A uniform magnetic field B fills a cylindrical volume velocity of the rod is half the terminal velocity.
radius R. A metal rod CD of length l is placed inside
the cylinder along a chord of circular cross-section as Q.21 A magnetic field B=(B0y/a) K̂ is into the plane
shown in the Figure. If the magnitude of magnetic field of paper in the +z direction. B0 and a are positive
increases in the direction of field at a constant rate constants. A square loop EFGH of side a, mass m
dB/dt, find the magnitude and direction of the E.M.F. and resistance R, in x-y plane, starts falling under the
induced in the rod. influence of gravity. Note the directions of x and y axes
in the Figure. Find

 O x
B
E F
b g
G H
C D
y
P hysi cs | 22.49

(i) The induced current in the loop and indicated its µ0 er 2


direction, (C) α (D) none of these
4 πR
(ii) The total Lorentz force acting on the loop and
indicated its direction, Q.2 A closed planar wire loop of area A and arbitrary
(iii) An expression for the speed of the loop, ν (t) and shape is placed in a uniform magnetic field of
its terminal value. magnitude B, with its plane perpendicular to magnitude
to magnetic field. The resistance of the wire loop is R.
The loop is now turned upside down by 180o so that
Q.22 A square loop of ‘a’ with a capacitor of capacitor its plane again becomes perpendicular to the magnetic
C is located between two current carrying long parallel field. The total charge that must have flowed through
wires as shown. The value of I is given as I=I0 sin ω t . the wire in the process is
(A) <AB/R (B) =AB/R (C) =2AB/R (D) None
a a
Q.3 A square loop of side a and resistance R is moved in
I I the region of uniform magnetic field B (loop remaining
a completely inside field), with a velocity v through a
distance x. The work done is:
(a) Calculate maximum current in the square loop.
B2 vx 2B2 2 vx 4B2 2 vx
(b) Draw a graph between charge on the lower plate of (A) (B) (C) (D) None
R R R
the capacitor v/s time.
Q.4 A metallic rod of length L and mass M is moving
Q.23 A long solenoid of radius a and number of turns under the action of two unequal forces F1 and F2
per unit length n is enclosed by cylindrical shell of radius (directed opposite to each other) acting at its ends
R, thickness d(d<<R) and length L. A variable current along its length. Ignore gravity and any external
i=i0 sin ω t flows through the coil. If the resistivity of the magnetic field. If specific charge of electrons is (e/m),
material of cylindrical shell is ρ , find the induced current then the potential difference between the ends of the
in the shell. rod is steady state must be
R
(A) |F1-F2|mL/eM (B) (F1-F2)mL/eM
a

(C) [mL/eM] ln [F1/F2] (D) None
d

Q.5 A rod closing the current (shown in Figure) moves


along a U shaped wire at a constant speed v under the
L action of the force F. The circuit is in a uniform magnetic
perpendicular to the plane. Calculate F if the rate of
heat generation in the circuit is Q.

F
Exercise 2
Single Correct Choice Type
Q v
(A) F=Qv (B) F= (C) F= (D) F= Qv
Q.1 An electron is moving in a circular orbit of radius v Q
R with an angular acceleration α. At the center of the
orbit is kept a conducting loop of radius r,(r<<R). The Q.6 Two parallel long straight conductors lie on a
e.m.f. induced in the smaller loop due to the motion of smooth surface. Two other parallel conductors rest
the electron is on them at right angles so as to form a square side
(A) Zero, since charge on electron in constant a initially. A uniform magnetic field B exists at right
angles to the plane containing the conductors. They all
µ0 er 2 start moving out with a constant velocity v. If r is the
(B) α
4R
2 2 . 5 0 | Electromagnetic Induction and Electromagnetic Waves

resistance per unit length of the wire the current in the Multiple Correct Choice Type
circuit will be
Bv Br Q.11 A conducting wire is placed in a magnetic field
(A) (B) (C) Bvr (D) Bv
r v which is directed into the paper. The magnetic field is
increasing at a constant rate. The directions of induced
Q.7An equilateral triangle loop ADC of some finite B currents in wire AB and CD are
as shown in the Figure. At time t=0, side DC of loop
is at edge of the magnetic field. Magnetic field is
perpendicular to the paper inwards (or perpendicular
to the plane of the coil). The induced current versus
time graph will be as

A
Figure
(A) B to A and D to C (B) A22.104
to B and C to D
(C) A to B and D to C (D) B to A and C to D

D C Q. 12 Two circular coils P& Q are fixed coaxially & carry


V currents I1 and I2 respectively
i i
I1 I2
(A) (B)
t t
P Q
i i
(A) If I2=0& P moves towards Q, a current in the same
(C) (D) direction as I1 is induced in Q
t t
(B) If I1=0 & Q moves towards P, a current in the opposite
direction to that of I2 is induced in P.
Q.8 A ring of resistance 10 Ω, radius 10cm and 100
turns is rotated at a rate 100 rev/s about its diameter is (C) When I1 ≠ 0 and I2 ≠ 0 are in the same direction
perpendicular to a uniform magnetic field of induction then the two coils tend to move apart.
10mT. The amplitude of the current in the loop will be (D) When I1 ≠ 0 and I2 ≠ 0 are in opposite directions
nearly (take: π2 =10 ) then the coils tends to move apart.
(A) 200A (B) 2A
(C) 0.002 A (D) None of these Q.13 A conducting rod PQ of length L= 1.0 m is moving
with a uniform speed v=20 m/s in a uniform magnetic
field B=4.0T directed into the paper. A capacitor of
Q.9 A long solenoid of N turns has a self-inductance
capacity C= 10 µF is connected as shown in Figure. Then
L and area of cross section A. When a current I flows
P
through the solenoid, the magnetic field inside it has
magnetic B. the current I is equal to:
A
(A) BAN/L (B) BANL B

(C) BN/AL (D) B/ANL
Q

Q.10 A small square loop of wire of side l is placed (A) qA=+800 µC andqB =
−800µC
inside a large square loop of wire of side L (L>>l).
(B) qA =
−800 µ C andqB =
+800µ C 
The loop are co-planner & their centers coincide. The
mutual inductance of the system is proportional to: (C) qA= 0= qB

l l2 L L2 (D) charged stored in the capacitor increases


(A) (B) (C) (D) exponentially with time
L L l l
P hysi cs | 22.51

Q.14 The e.m.f. induced in a coil of wire, which is (A) There will be repulsion between A and B if i is
rotating in a magnetic field, does not depend on increased
(A) The angular speed of rotation (B) There will be attraction between A and B if i is
increased
(B) The area of the coil
(C) There will be neither attraction nor repulsion when
(C) The number of turns on the coil
i is changed
(D) The resistance of the coil
(D) Attraction or repulsion between A and B depends
on the direction of current. It does not depend whether
Q.15 Current growth in two L-R circuit (b) and the current is increased or decreased.
(c) as shown in Figure (a). Let L1,L2, R2 and R2 be the
corresponding value in two circuits, then
Q.19 A bar magnet is moved along the axis of copper
ring placed far away from the magnet. Looking from
i the side of the magnet, an anticlockwise current is
(b) L1 ( c ) R1 L12 R21 found to be induced in the ring. Which of the following
(b) (c) may by true?
t V S V S (A) The south pole faces the ring and the magnet moves
towards it.
(a) (b) (c)
(B) The north pole faces the ring and the magnet moves
(A) R1 > R 2    (B) R1 = R 2   (C) L1 > L2 (D) L1 < L2
towards it.
(C) The south pole faces the ring and the magnet moves
Q.16 The dimension of the ratio of magnetic flux and
away from it.
the resistance is equal to that of:
(D) The north pole faces the ring and the magnet moves
(A) Induced e.m.f. (B) Charge
away from it.
(C) Inductance (D) Current
Q.20 AB and CD are smooth parallel rails, separated
Q.17 Figure 22.73 shows a P by a distance l, and inclined to the horizontal at an
plane figure made of a angel θ. A uniform magnetic field of magnitude B,
conductor located in a Q directed vertically upwards, exists in the region. EF is a
magnetic field along the conductor of mass m, carrying a current i. For EF to be
inward normal to the plane R in equilibrium,
of the figure. The magnetic B F D
field starts diminishing.
Then the induced current
C  B
(A) At point P is clockwise
(B) At point Q is anticlockwise 
E
(C) At point Q is clockwise

(D) At point R is zero A
(A) i must flow from E to F (B) Bil=mg tan θ
Q.18 Two circular coils A and B are facing each
other as shown in Figure. The current I through (C) Bil=mg sin θ (D) Bil=mg
A can be altered
A B Q.21 In the previous question, if B is normal to the
plane of the rails
(A) Bil=mg tan θ
(B) Bil=mg sin θ
(C) Bil=mg cos θ
i

 (D) equilibrium cannot be reached


2 2 . 5 2 | Electromagnetic Induction and Electromagnetic Waves

Q.22 A semicircle conducting ring of radius R is placed Q.25 Two metallic rings A and B, identical in shape
in the xy plane, as shown in the Figure. A uniform and size but having different resistivities ρA and ρB ,
magnetic field is set up along the x-axis. No net e.m.f., are kept on top of two identical solenoids as shown in
will be induced in the ring. If the Figure. When current I is switched on in both the
Y solenoids in identical manner, the rings A and B jump to
heights hA and hB respectively, with hA>hB. The possible
relation (s) between their resistivity and their masses mA
B
and mB is (are)
R

X A B

(A) It moves along the x-axis


(B) It moves along the y-axis
(C) It moves along the z-axis
(D) It remains stationary

Q.23 In the given diagram, a line of


(A) ρA > ρB and mA =mB (B) ρA < ρB and mA =mB
force of a particular force field is
shown. Out of the following options, (C) ρA > ρB and mA > mB (D) ρA < ρB and mA < mB
it can never represent
(A) An electrostatic field Assertion Reasoning Type
(B) A magnetic field
Q.26 Statement-I: A vertical iron rod has a coil of wire
(C) A gravitation field of mass at rest wound over it at the bottom end. An alternating current
(D) An induced electric field flows in the coil. The rod goes through a conducting
ring as shown in the Figure. The ring can float at a
certain height above the coil because
Q.24 The Figure shows certain wire segments joined
together to form a coplanar loop. The loop is placed in
a perpendicular magnetic field in the direction going
into the plane of the figure. The magnitude of the field
increases with time.I1 and I2 are the currents in the
segments ab and cd. Then,

c d
a b

Statement-II: In the above situation, a current is


induced in the ring which interacts with the horizontal
component of the field to produce an average force in
the upward direction.
(A) Statement-I is true, statement-II is true; statement-II
(A) I1 > I2 is a correct explanation for statement-I.
(B) I1 < I2 (B) Statement-I is true, statement-II is true; statement-II
is not a correct explanation for statement-I.
(C) I1 is in the direction ba and I2 is in the direction cd
(C) Statement-I is true, statement-II is false
(D) I1 is in the direction ab and I2 is in the direction dc
(D) Statement-I is false, statement-II is true
P hysi cs | 22.53

Comprehension Type Q.30 The value of magnetic flux in each case is given by

Comprehension-I ( A ) Case I : Φ = π (L2 + 2 ) B


A conducting ring of radius a is rotated about a point (
Case II : Φ = π L2 − 2 B )
(B ) Case I : Φ = π (L )
O on its periphery as shown in the Figure on a plane 2 2
+ B
perpendicular to uniform magnetic field B which exists
everywhere. The rotational velocity is w. Case II : Φ = π (L +  ) B
2 2

P B ( C ) Case I : Φ= (L +  ) B
2 2

Case II : Φ= (L −  ) B
2 2

(D ) Case I : Φ= (L +  ) B
2
O Q
Case II : Φ = π (L −  ) B
2

R
Q.31 The direction of induced current in the case I is
Q.27 choose the correct statement (s) related to the
potential of the points P, Q and R (A) From a to b and from c to d

(A) VP − V0 > 0 and VR − V0 < 0 (B) From a to b and from f to e

(B) V= (C) From b to a and from d to c


P VR > V0

(C) (D) From b to a and from e to f


V0 > VP =
VQ

(D) VQ − VP = VP − V0
Q.32 The direction of induced current in the case II is

Q.28 Choose correct statement (s) related to the (A) From a to b and from c to d
magnitude of potential differences (B) From b to a and from f to e
1 1 (C) From b to a and from c to d
(A) VR − V0 = Bωa2 (B) VP − VQ = Bωa2
2 2
(D) From a to b and from d to c
2 2
(C) VQ − VO =2Bωa (D) VP − VR =2Bωa

Q.33 If I1 and I2 are the magnitudes of induced current
in the cases I and II, respectively, then
Q.29 Choose the correct statement(s) related to the
induced current in the ring (A) I1 =I2 (B) I1 > I2   

(A) Current flows from Q → P → O → R → Q (B) (C) I1 < I2   (D) Nothing can be said
Current flows from Q → R → O → P → Q
(C) Current flows from Q.34 Match the Following Columns

Q → P → O and Q → R → O Column 1 Column 2


(D) No current flows (A) Dielectric (P) Time independent
ring uniform electrostatic field
Comprehension–II The adjoining Figure 22.80 shows charged out of system
two different arrangements in which two square wire (B) Dielectric (Q) Magnetic field
frames of same resistance are placed in a uniform ring uniform
constantly decreasing magnetic field B. charged
g Rotating with
h h g angular velocity.
e e f
f (C) Constant (R) Induced electric filed
current i0 in ring
c d d l c
b l b (D) Current i=i0 cos (S) Magnetic moment
a a
L L ω t in ring
I II
2 2 . 5 4 | Electromagnetic Induction and Electromagnetic Waves

Previous Years’ Questions (C) I1 is in the direction ba and I2 is in the direction cd

Q.1 An infinitely long cylinder is kept parallel to a (D) I1 is in the direction ab and I2 is in the direction dc
uniform magnetic field B directed along positive z-axis.
The direction of induced as seen from the z-axis will be Q.3 A thin flexible wire of length L is connected to
 (2005) two adjacent fixed points and carries a current l in the
clockwise direction, as shown in the Figure. When the
(A) Clockwise of the+ ve z-axis
system is put in a uniform magnetic field of straight B
(B) Anticlockwise of the + ve z-axis going into the plane of the paper, the wire takes the
(C) Zero shape of a circle. The tension in the wire is  (2010)

(D) Along the magnetic field

Q.2 The Figure shows certain wire segment joined


together to from a coplanar loop. The loop is placed in
a perpendicular magnetic field in the direction going
into the plane of the figure. The magnitude of the field.
The magnitude of the field increases with time. I1 and
I2 are the currents in the segments ab and cd. Then,
 (2009)
IBL IBL IBL
c d (A) IBL (B) (C) (D)
π 2π 4π
a b
Q.4 You are given many resistances, capacitors and
inductors. These are connected to a variable DC voltage
source (the first two circuits) or an AC voltage source
of 50Hz frequency (the next three circuits) in different
ways as shown in column II. When a current I (steady
state for DC or rms for AC) flows through the circuit, the
corresponding voltage V1 and V2 (indicated in circuits)
(A) I1 > I2 are related as shown in column I. (2010)
(B) I1 > I2

Column I Column I
V1 V2
(A) I ≠ 0, V1 is proportional to I

6mH 3F
(p)

V
V1 V2
(B) I ≠ 0, V2 > V1

6mH 2
(q)

V
P hysi cs | 22.55

V1 V2
(C)=
V1 0,
= V2 V

6mH 2
(r)

V1 V2
(D) I ≠ 0, V1 is proportional to I

6mH 3F
(s)
V
V1 V2

1k 3F
(s)
V

Passage I Q.6 After the capacitor gets fully charged, S1 is opened


and S2 is closed that the inductor in series with the
The capacitor of capacitance C and be charged (with
capacitor. Then, (2006)
the help of a resistance R) by a voltage source V, by
closing switch S2 open. The capacitor can be connected (A) At t=0, energy stored in the circuit is purely in the
in series with an inductor L by closing switch S2 and form of magnetic energy
opening S1 (See fig.).
(B) At any time t>0, current in the circuit is in the same
V direction
(C) At t>0, there is no exchange of energy between the
R
C S1 inductor and capacitor
(d) At any time t>0, maximum instantaneous current in
c
S2 the circuit may be V
L

Q.7 If the total charge stored in the LC circuit is Q0, then


Q.5 Initially, the capacitor was uncharged. Now, switch
for t ≥ 0  (2006)
S1 is closed and S2 is kept open. If time constant of this
circuit is t, then  (2006) π t 
=
(A) The charge on the capacitor is Q Q0 cos  + 
(A) After time interval t, charge on the capacitor is CV/2 2 Lc 
(B) After time interval 2t, Charge on the capacitor is CV π t 
(1-e-2) =
(B) The charge on the capacitor is Q Q0 cos  − 
2 Lc 
(C) The work done by the voltage source will be half of
the heat dissipated when the capacitor is fully charged d2Q
(C) The charge on the capacitor is Q = − LC
(D) After time interval 2t, charge on the capacitor is CV dt2
(1-e-1) 2
(D) The charge on the capacitor is Q = − 1 d Q
2
LC dt
2 2 . 5 6 | Electromagnetic Induction and Electromagnetic Waves

Q.8 Two different coils have self-inductances L1=8 mH (B) The emf induced in the loop is finite if the current
and L2 =2 mH. The current in one coil is increased at is constant
a constant rate. The current in the second coil is also
(C) The emf induced in the loop is zero if the current
increased at the same constant rate. At a certain instant
decreases at a steady rate
of time, the power given to the coils is the same, At that
time, the current the induced voltage and the energy (D) The emf induced in the loop is finite if the current
stored in the first soil are i1,V1 and W1 respectively. decreases at a steady rate
Corresponding value for the second coil at the same
instant are i2, V2 and W2 respectively. (1994) Q.12 If the direct transmission method with a cable of
Then resistance 0.4 Ω km−1 is used, the power dissipation (in
i1 i1 %) during transmission is  (2013)
1 W1 1 V1
(A) = (B) = 4 (C) = (D) =4
i2 4 i2 W2 4 V2 (A) 20 (B) 30 (C) 40 (D) 50

Q.13 In the method using the transformers, assume


Q.9 A series R-C circuit is connected to AC voltage
that the ratio of the number of turns in the primary to
source. Consider two cases; (A) when C is without a
that in the secondary in the step-up transformer is 1 :
dielectric medium and (B) when C is filled C is filled
10. If the power to the consumers has to be supplied at
with dielectric of constant 4. The current IR through the
200 V, the ratio of the number of turns in the primary to
resistor and VC across the capacitor are compared in the
that in the secondary in the step-down transformer is
two cases. Which of the following is/are true?  (2011)
 (2013)
A B A B A B A B
(A) IR > IR (B) IR < IR (C) VC > IC (D) VC < IC (A) 200 : 1 (B) 150 : 1 (C) 100 : 1 (D) 50 : 1

Q.14 A conducting loop in the shape of a right angled


Q.10 A circular wire loop of radius R is placed in the x-y
isosceles triangle of height 10 cm is kept such that the
plane centered at the origin O. A square loop of side
90o vertex is very close to an infinitely long conducting
a (a < < R) having two turns is placed with its centre
wire (see the figure). The wire is electrically insulated
at z = 3 R along the axis of the circular wire loop, as from the loop. The hypotenuse of the triangle is parallel
shown in figure. The plane of the square loop makes to the wire. The current in the triangular loop is in
an angle of 45o with respect to the z-axis. If the mutual counterclockwise direction and increased at a constant
µ0 a2 rate of 10 A s-1. Which of the following statement(s)
inductance between the loops is given by , then
21/2 R is(are) true? (2016)
the value of p is  (2012)
z
o
90
o 10 cm
45

 µ0 
 (A) The magnitude of induced emf in the wire is  
3R volt  π 

(B) If the loop is rotated at a constant angular speed


R  µ0 
y about the wire, an additional emf of   volt is
O
induced in the wire  π 
(C) The induced current in the wire is in opposite
direction to the current along the hypotenuse
Q.11 A current carrying infinitely long wire is kept along
the diameter of a circular wire loop, without touching it. (D) There is a repulsive force between the wire and the
The correct statement(s) is (are) (2012) loop

(A) The emf induced in the loop is zero if the current is


constant.
P hysi cs | 22.57

Q.15 Two inductors L1 (inductance 1 mH, internal Which of the following schematic plot(s) is(are) correct?
resistance 3 Ω) and L2 (inductance 2 mH, internal (Ignore gravity)
resistance 4 Ω), and a resistor R (resistance 12 Ω) are all
I(x) F(x)
connected in parallel across a 5V battery. The circuit is
switched on at time t=0. The ratio of the maximum to
the minimum current (Imax/Imin) drawn from the battery (A) (B) L
is  (2016) x x
0 L 2L 3L 4L 0 2L 3L 4L

Q.16 A rigid wire loop of square shape having side of


length L and resistance R is moving along the x-axis I(x)
with a constant velocity v0 in the plane of the paper. v(x)
At t=0, the right edge of the loop enters a region of v0
length 3L where there is a uniform magnetic field B0 3L 4L
x
(C) (D)
into the plane of the paper, as shown in the figure. 0 L 2L x
For sufficiently large v0, the loop eventually crosses 0 L 2L 3L 4L
the region. Let x be the location of the right edge of
the loop. Let v(x), I(x) and F(x) represent the velocity
of the loop, current in the I(x)
loop, and force on the loop, F(x)
respectively, as a function of x. Counter-clockwise
current is taken as positive. (2016)
(A) (B) L
x x
0 L 2L 3L 4L 0 2L 3L 4L

I(x)
L v(x)
v0 v0
3L 4L
(C)
0 xx (D)
0 L 2LL 2L
3L 4L x
0 L 2L 3L 4L

MASTERJEE Essential Questions

JEE Main/Boards JEE Advanced/Boards

Exercise 1 Exercise 1
Q.4 Q.7 Q.9
Q.5 Q.8 Q.9
Q.13 Q.14 Q.19
Q.18
Q.20

Exercise 2 Exercise 2
Q.3 Q.4 Q.7
Q. 1 Q.2 Q.8
Q.13 Q. 15 Q.24
Q.11 Q.14
Q.25 Q.26
2 2 . 5 8 | Electromagnetic Induction and Electromagnetic Waves

Answer Key

JEE Main/Boards Q.12 µr =1000

Q.13 Along PSRQP


Exercise 1
Q.16 emax=0.6032 V and eav=0
Q.2 0.4 V
Q.18 F=0.00375 N
Q.3 8 × 10 −5 Wb. 8 × 10 −3 Wb
Q.20 (i) Mutual inductance
Q.5 5.03 × 10 −4 H
(ii) The current product in coil B depends on:
Q.6 6.25 × 10−3 H
(a) Number of turns in the coil
Q.7 0.1 H
   (b) Natural of material
Q.8 Clockwise Direction    (c) geometry of coil
Q.9 2.56 V Q.21 (ii) 625 × 10 −4 V
3
Q.10 By decreasing current from 2 A to zero in 0.28s
Q.24 (i) Same
Q.11 1.58 × 10 −4 H
(ii) Current in copper loop is more than aluminum loop

Exercise 2

Single correct choice type

Q.1 C Q.2 A Q.3 A Q.4 C Q.5 A Q.6 D

Q.7 A Q.8 A Q.9 B Q.10 C Q.11 A Q.12 A

Q.13 C Q.14 A Q.15 D Q.16 D Q.17 B Q18 B

Q.19 D Q.20 C Q.21 D Q.22 A

Previous Years’ Question


Q.1 D Q.2 D Q.3 B Q.4 B Q.5 D Q.6 B

Q.7 D Q.8 D Q.9 D Q.10 D Q.11 C Q.12 D

Q.13 B Q.14 D Q.15 A Q.16 B Q.17 A Q.18 D

Q.19 C Q.20 D Q.21 A Q.22 C Q.23 D Q.24 C

JEE Advanced/Boards
Exercise 1
Q.1 10 µV Q.2 λVyB0

Q.3 5.0 µV Q.4 2N


2
µ0ia π Rmg
Q.5 Q.6
2Rb B2  2
P hysi cs | 22.59

1
Q.7 A Q.8 0.4V
3
Q.9 3 µV ,clockwise
erk
Q.10 directed along tangent to the circleof radius r, whose center lies on the axis of cylinder
2m
πa2C
Q.11 Q.12 200 rad/s
R
Q.13 (i) 85.22 Tm2; (ii)56.8 V (iii) Linearly Q.14 (i) 2.4 × 10−5 V (ii) from c to b

Q.15 (i) 3.3 × 108 A, (ii) 4.1 × 107 W, (iii) totally unrealistic Q.16 
E w s
3w 4w

 s
F
s

2
µ0ihωimN b  dB 2 
Q.17 ln Q.18 R −
2π a 2 dt 4

7 3 1 1 Bωr 2 1 − e−Rt/L 
Q.19
= iEA = Al; iBE = A; iFE A 2
Q.20 (i) E= Bωr (ii) I =  
22 11 22 2 2R
mgR g
Q.21 (i) Vterminal = ;(ii)
2 2
BL 2
µ0 a
CI0 ω2 ln2,
( µ0ni0 cos ωt ) πa2 (Ld)
Q.22 (a) Imax= Q.23 I =
π ρ2πR

(b) Qo
  
/ 2/
t
/2 3/2
-Qo

Exercise 2

Single Correct Choice Type

Q.1 B Q.2 C Q.3 D Q.4 A Q.5 B Q.6 A

Q.7 B Q.8 B Q.9 A Q.10 B

Multiple Correct Choice Type

Q.11 A Q.12 B, D Q.13 A Q.14 D Q.15 B, D Q.16 B

Q.17 A, B, D Q.18 A Q.19 B, C Q.20 A, B Q.21 B Q.22 A, B, C, D

Q.23 A, C Q.24 D Q.25 B, D


2 2 . 6 0 | Electromagnetic Induction and Electromagnetic Waves

Assertion Reasoning Type

Q.26 C

Comprehension Type

Q.27 B, D Q.28 A, C Q.29 D Q.30 C Q.31 C Q.32 B

Q.33 B

Match the Column Type

Q.34 A → P; B → P, Q, S; C → Q, S; D → Q, R, S

Previous Years’ Questions


Q.1 C Q.2 D Q.3 C Q.5 B Q.6 D Q.7 C

Q.8 A, C, D Q.9 B, C Q.10 7 Q.11 A, C Q.12 B Q.13 A

Q.14 A, D Q.15 8 Q.16 C, D

Solutions

JEE Main/Boards di
N∈=– M
dt
Exercise 1 m0nNA=M

M=4p×10–7 × 50 × 102 × 200 × 4 × 10–4 =5.03 × 10–4 H


Sol 1: No, as the voltmeter also gets induced emf.
∆I
dφ nB dA Sol 6 : ε = M
Sol 2: E.m.f.= = ∆T
dt dt
(∆A=2A as it turned through 180o) 4
50 × 10–3= M.
1
4 × 10 –4 × 103 × 500 × 10 –4 × 2 2
= = 0.4V
1
50 × 10 –3
10 M= =6.25 × 10–3H
8
dφ di
Sol 3: =L ∆I
dt dt Sol 7: ε = L
∆T
φ=Li; φ=20 × 10–3 × 4 × 10–3=80 µ Wb 4
4 × 104= L .
Total flux=hφ=100 × 80 mWb=8000 mWb 10 × 10 –6
L=0.1 Henry
Sol 4: Field is perpendicular outwards the paper. As
the loop area increases, net flux increases, so induced µ0i
Sol 8: B=
current tries to reduce flux. So it flow clock wise. 2πr
dφ=B.dA=B.dr
Sol 5: B=m0ni
µ 0 i
dB di dφ= dr
ε =– A =– m0nA 2πr
dt dt
P hysi cs | 22.61

µ 0 i r2 4 × π × 10 –7 × 10 × 0.2 0.1 ⇒Di increases. i increases


φ= ln = ln
2π r1 2π 0.05 \bulb becomes brighter

dφ 2.77 × 10 –7 After completely removing, the current again decreases


ε= = =1.39 × 10–5 v = 2.77 × 10–7Wb.
dt 2 × 10 –2 V
as steady state current is I0= , which was also initial
current r
Current will be in clockwise direction.

∆A Sol 15: The voltage induced across a conductor when


Sol 9: ε = Bn (∆A=2A as it turns 180º)
∆T it is exposed to a varying magnetic field in called
Bn2A 0.4 × 100 × 2 × (8 × 10 –2 )2 inducedemf.
= = = 2.56V
t 0.2 dφ
ε=–
∆i dt
Sol 10: ε = –L
∆t ⇒ dV=– B v(d )
(–2) ⇒ dV=– Bω r dr
50 = –5
∆t BωL2
V=
T=0.28 2

Current should reduce to 0 in 0.28.


Sol 16: A=A0 cos ωt

Sol 11: B=m0n1i φ=BnA = nB.A0 cos wt

di dφ
ε =– L e=– =nBA0 w sin wt
dt dt

di = 20 × 3 × 10–2 × π(8 × 10–2)2 × 50=0.6 V


 d   d 
ε = n2  – B.A  = n2  – µ0 in1 πr 2  = n2m0n1pr2 dt ⇒ eav=0 as in one complete rotatior, Se= 0
 dt   dt 
1000
⇒ M=m0 n1n2pr2 = 4p×10–7× ×100×π(2×10–2)2 Sol 17: If a current i in a coil changes with time on e.m.f.
100 × 10 –1
= 1.58 × 10 H –4 is induced in the coil. The self – induced rmf is ∈L=– L
di
Sol 12: L ∝ m dt

L2 µ2 B=m0 in
= di
L1 µ1 ε =– m0npr2 dt ∴ L=m0npr2

m1=m0, m2=mrm0
N µ Nπr 2
L2
10 π= ∴ L= 0
⇒ mr= = =1000  
L1 0.01
\ mr=1000 Sol 18: ε =Blv
B V
Sol 13: It flows anti-clock wise to increase flux along i=
R
outside the plane. Hence it flow PSRQP. B2 2 V (0.15)2 (0.5)2 (2)
F=iB = = =3.75×10–3 N
R 3
di
Sol 14: ε = –L
dt Sol 19: The currents induced in a solid conducting
Solenoid tries to go back to initial state i.e. If an action body as it passes through a magnetic field is called
produce a change Dε1, solenoid tries to produce a eddy current.
change Dε2 such that Dε2 is in Opposite direction of Dε1. Eddy currents lead to heating up of Transformer core.
When you remove iron core, L keeps decreasing Eddy current is reduced by making transformer with
thin slabs.
2 2 . 6 2 | Electromagnetic Induction and Electromagnetic Waves

Exercise 2
Sol 1 : (C) A=pR2 =π(R0 + t)2
dA
=2π (R0 + t)
dt
–BdA
ε= =– 2πB(R0 + t)
dt
Sol 20: (i) The Principal involved is mutual inductance \2π(R0 + t)B is induced anticlockwise.
(ii) The current produced in coil B depends on Note: To have clarity about clockwise or anticlockwise,
(a) number of turns in the coil, remember as flux increases, it tries to reduce net
magnetic field B. Hence voltage is induced. It leads to
(b) Nature of material current in direction of voltage, which reduces magnetic
(c) geometry of coil field.

BA
Sol 21: (i) Faraday’s law of electromagnetic induction Sol 2 : (A) E=
∆t
An emf is induced in the loop when the number
of magnetic field lines that pass through the loop is 20 × (0.1)2
10=
changing. ∆t
(ii) ε =Blv tanθ
∴ Dt=20 ms
5 1
=5×10 ×25×1800×
–4
×
18 3 Sol 3 : (A) [MA–1T–2]
625
= ×10 V
–4

3 µ0i
Now B= (for circular wire)
2r
Sol 22: (a) The current induced in a solid conducting
body as it passes through a magnetic field is called eddy [B][r] [MA –1 T –1 ][L]
⇒ [m0]= = = MLA–2T–2
current. It is used in induction stove, water heaters, etc. [ i] [A]
(b) (i) ∈=Blv
Sol 4 : (C) Induced emf tries to push the coil upward in
∈ B v B2  2 v
(ii) i= = ; F=iB= case II and magnet in case-I, to present sudden change
R R R
in net flux.
∈2 B2 2 v 2
(iii) Power dissipated P= = \a1, a2< g
R R

Sol 23: If a current i in a coil charges with time, an emf Sol 5 : (A) For a circular loop B at center is greater than
B at any point along the axis.
di
is induced in the coil. The self-inducedemf is ∈L=– L
dt When both the loops approach each other, magnetic
S.I unit Henry–H. field (B) starts increasing at center. To compensate it,
Current decreasing.
Sol 24: (i) Inducedemf is same →
Sol 6 : (D) Let the triangle travel a distance x along v
ε = 2pr2w B in time t.
(ii) Current in copper is more, as its resistance is less. Area of triangle in magnetic field
1
Sol 25: It induces current in opposite direction. A= x(2x) = x2
2
A=v2t2
Sol 26: Emf induces Anticlockwise as seen from north.
Both Magnets produce current in same direction. –BdA
E=
dt
P hysi cs | 22.63

d 2 2 Sol 13 : (C) Let displacement of PQ be x.dx be small


iR= B (v t )
displacement along dv
dt
2Bv 2 dE=vBdx
i= t
R v=xω
\i∝t \dE=ωB xdx
2
→ → WB 2 x0 ωBx0
Sol 7 : (A) E=  v × B  .  ⇒ E= x ⇒ E=
  2 0 2
 
x20 =l2 + L2
 ( ) (  ) ( )
=  2iˆ + 3ˆj + Kˆ × ˆi + 2ˆj  2kˆ =  –2iˆ + ˆj + kˆ  2kˆ 
  
E=2V \E=
(
ωB L2 + 2 )
2
→ →
    ( )( )
Sol 8 : (A) E=  V× B  . l= 10iˆ × 4kˆ  0.3ˆj = 12 V
Sol 14 : (A) Current is from P to Q
 
(A)
(
Sol 9 : (B) E= V × B . l ) π π
µ0 I Sol 15 : (D) wt= ⇒ t=
B= 2 2ω
2πr
µ0 I BDA B A 2ωBA
E= . V. l Avg. E.m.f= = =
2πr Dt π π
ω
→ 2
Sol 10 : (C) B=B K ; V= v x ˆi + v y ˆj ; L= 1ˆi +  2ˆj
Sol 16 : (D) dε = vB dl


ˆ (ˆ )
E=(v × B). () = Vx i + Vy j × Bk .  – ˆi +  2ˆj
ˆ 
  
V = lw
ˆ ˆ
= + Vx B, j + v yBl2 i \d∈=Bwldl
2
⇒ VA – VB=VyBl2 B ω 2
E= Bω∫ d =
2
VC – VB=Vx Bl1 1

L
VA – VC=VyBl2 –VxBl1 l2=L; l 1=
2
\VA – VC∝ (Vxl2 – Vyl1)
Bω  2  L   3Bω2
2
\Î= L –  =
2   2   8

Sol 11 : (A) V =2i
=5 cos θ î + 5 sin θ ˆj =3 î + 4 ˆj Sol 17 : (B) Electric field is induced to left
→ → \it accelerates to right (B)
E=  V× B  . = 2iˆ × (3ˆj + 4kˆ  3iˆ + 4ˆj
     
 
µ0i
Sol 18 : (B) B=
= 6kˆ – 8ˆj 3iˆ + 4ˆj = 32 Volts 2R
  
dB µ0 di
=
Sol 12 : (A) φ=B.dA dt 2R dt
– µ0 di
x 2 E= . pr2
φ= B0 1 + d 2R dt
a

dφ dφ dx di
E= –L
= . dt
dt dv dt

B0 d2 µ0 πr 2
⇒ L=
= V 0. 2R
a
2 2 . 6 4 | Electromagnetic Induction and Electromagnetic Waves

d Sol 4: (B) Magnetic field produced by a current I in a


Sol 19 : (D) E= – B.dA
dt large square loop at its centre,
B.dA=0 Q E=0 ⇒ L=0 i
B∝
Note: Simply we can say. The magnetic field vectors will L
be along the plant. i
Say B=K
L
B.dA=0
∴ Magnetic flux linked with smaller loop
\E=0
φ=B.S
⇒ L=0
 i
φ=  K  (l2)
Sol 20 : (C) Current increases  L
Therefore, the mutual inductance
⇒ magnetic field increases at a giver point. Magnetic
field also decreases radially. Hence to nullify the φ 2 2
M= =K or M ∝
increases magnetic field, loop B repels. i L L
Note Dimensions of self-inductance (L) or mutual
Sol 21 : (D) φ= ∫ B.dA =K inside loop, – K outside loos inductance (M) are:
\total φ=0 [Mutual inductance]=[Self-inductance]=[m0][length]
Similarly dimensions of capacitance are :
Sol 22 : (A) In (a), magnetic field in perpendicular to
[capacitance]=[e0][length]
plane others along plane,
From this point of view options (b) and (d) may be
∴ in others it is minimum, maximum in (a)
correct

Previous Years’ Questions Sol 5 : (D) The current-time (i – t) equation in L-R circuit
is given by [Growth of current in L-R circuit]
Sol 1 : (D) Net change in magnetic flux passing through –t/tL
the coil is zero. i=i0( 1 – e ) … (i)

∴ Current (or emf) induced in the loop is zero


V 12
whereiL= = =2 A
R 6

Sol 2 : (D) Induced motional emf in MNQ is equivalent


L 8.4 × 10 –3
and i0= = = 1.4 × 10–3 S
to the motional emf in an imaginary wire MQ i.e., R 6
and i=1A(given), t=?
eMNQ=eMQ=Bvl=Bv(2R) [=MQ=2R]
Substituting these values in Eq. (i), we get
Therefore, potential difference developed across the
ring is 2RBv with Q at higher potential. t=0.97 × 10–3 s
or t=0.97 ms ⇒ t=1 ms
Sol 3 : (B) A motional emf, e=Blv is induced in the
rod. Or we can say a potential difference is induced
Sol 6 : (B)
between the two ends of the rod. AB with A at higher
potential and B at lower potential. Due to this potential →→
dϕ dB
difference, there is an electric field in the rod. ∫ E d = dt
=S
dt

dB
or E(2pr)=pa2 dt for r ≥ a

a2 dB
∴ E=
2r dt

1
Induced electric field ∝
r
P hysi cs | 22.65

For r ≤ a Sol 9: (D) When switch S is closed magnetic field lines


dB passing through Q increases in the direction from right to
E(2pr)=pr2 dt left. So according to Lenz’s law induced current in Q i.e.,
IQ will flow in such a direction, so that the magnetic field
1
lines duet to IQ passes from left to right through Q. This
r dB 1
Or E= or E ∝ r is possible when IQ flows in anticlockwise direction as
2 dt 1
seen by E. Opposite is the case when switch S is opened
a dB i.e., IQ will be clockwise as seen by E.
At r=a, E= 2
2 dt
Therefore, variation of E with r (distance from centre) e2
Sol 10 : (D) Power P=
will be as follows R
 dϕ 
Here, e=induced emf=–  
 dt 
whereφ=NBA

 dB 
E=– NA  
 dt 
1
Also, R ∝
r2
Sol 7: (D) The equations of l1(t), l2(t) and B(t) will take Where R= resistance, r=radius,
the following form : =length.
–k 2t
I1(t)=K1(1 – e ) → current growth in L-R circuit ∴ P ∝ N2r2
–k 2t
B(t)=K3(1– e ) → (t) ∝ I1(t) P2
∴ =4
P1
I2(t)=K4 e –k 2t

 e dI dI  Sol 11: (C) Direction of polarization is parallel to


= 2 and e2 ∝ 1 e=
I2 (t) 2 –m 1  magnetic field,
 R dt dt   
–k t
∴ X || B
Therefore, the product I2(t)B(t)=K5 e–k2t (1 – e 2 ). The  
value of this product is zero at t=0 and t=∞. Therefore, and direction of wave propagation is parallel to E × B
  
the product will pass through a maximum value. The ∴ K || E × B
corresponding graphs will be as follows :
Sol 12 : (D) Oscillating coil produces time variable
magnetic field. It cause eddy current in the aluminium
plate which causes anti–torque on the coil, due to
which is stops.

Sol 13 : (B) Charge on the capacitor at any time ‘t’ is


=q CV (1 − e− t/ τ )

At t = 2τ

=q CV (1 − e−2 )

2 1
Sol 14 : (D)
Sol 8 : (D) Electric field will be induced in both AD and r
BC.
R d
2 2 . 6 6 | Electromagnetic Induction and Electromagnetic Waves

Let M12 be the coefficient of mutual induction between


loops 
φ =M 12i2 Fl

µ0i2R 2 2
⇒ πr =
M12i2
2(d2 + R 2 )3/2 lg
µ0R 2 πr 2 Sol 22 : (C)
⇒ M12 =
2(d2 + R 2 )3/2 µ0
2πi
BA =
φ2= M12i1 ⇒ φ2= 9.1 × 10 −11 weber 4 π (  / 2π )
 µ0 i 
=BB  (sin 45o + sin 45o ) × 4
8 −9
= 6 V /m  4π  / 8 
Sol 15 : (A) E0 = CB0 =3 × 10 × 20 × 10
BA 2
π
=
BB 8 2
Sol 16 : (B)
Infrared waves → To treat muscular strain
Sol 23 : (D)
radio waves → for broadcasting
Radiation energy per quantum is
X-rays → To detect fracture of bones
E=hv
Ultraviolet rays → Absorbed by the ozone layer of the
atmosphere; As per EM spectrum, the increasing order of frequency
and hence energy is
Sol 17 : (A) Energy is equally divided between electric Radio wave < Yellow light < Blue light < X Ray
and magnetic field.
Sol 24 : (C) For electromagnet and transformer, the
Sol 18 : (D) Since resistance and inductor are in parallel, coercivity should be low to reduce energy loss.
so ratio will be 1.

Sol 19 : (C) When K1 is closed and K2 is open, JEE Advanced/Boards


E
I0 = Exercise 1
R
when K1 is open and K2 is closed, current as a function 4
Sol 1 : e=B tan θ × v .l =3 × 10–4× ×0.1×0.23
of time ‘t’ in L.R. circuit. 3
−R
t
1 −5 1 =10–5 . V=10mV
I = I0 e=L =e = 0.67 mA
10 1500
Sol 2 : E = (V × B).
Sol 20 : (D) V= v x ˆi + v y ˆi
1 2 1 2 1 2
As L1 > L2 , therefore L i > L > L 2i , B=– B0kˆ
2 1 2 2
∴ Rate of energy dissipated through R from L1 will be Here = λ î we us are taking a cross PQ
slower as compared to L2.
\E=[( v x ˆi + v y ˆj ) × B0kˆ ] (li)=λ VyB0

Sol 21 : (A)
 µ I2  Sol 3 : A=pr2
 0 
E  4 πL sin θ 
tan=
θ =   dA dA dr dr
= . =2pr.
λ g λ g dt dr dt dt

πλLg –BdA
⇒ I = 2 sin θ E ⇒=–
µ0 cos θ dt
P hysi cs | 22.67

dr Sol 7 :
=– B2pr . =(0.02) . 2π (4 × 10–2) . (1 × 10–3) =5 mV
dt

Sol 4 : Consider the sum as two loops, one with 6r and


other 3r.
E=BLV =2 × 1 × 2=4V
Similarly in loop with 3r also 4 V is induced.
Let AB be diameter rod, CD be external resistor CD is
hence the circuit can be shown as
fixed
R2
area of part I be A1= θ
2
dA BR 2 dθ
E= B =
dt 2 dt
BR 2 w 500 × (0.1)2 × 20
E= = =5 V
2 2
4 R
⇒ 1= = 1A 20
1 RAC= AB = =10 W
2+ 2 2
1 1
+
6 3
⇒ F=ilb = 1 × 1 × 2 = 2 N

µ0i µ0i
Sol 5 : B= =
2r 2b
Area of small coil Ai=pa2
5 1
d
2
d πa .µ0i i= = A
∈= BA = 1 3
dt dt 2b 10 +
1 1
+
πa2µ0 di 10 10
∈= 1
2b dt Current through external resistance is A
3
dQ
∈ =iR = R
dt dB 3 2 dB 3
Sol 8 : E= A = a . = (a)2. 3
2 dt 4 dt 4
dQ πa µ0 di
⇒R = E=3V
dt 2b dt
Emf induced is 3V
πa2µ0 πa2µ0
⇒DQ= ∆i = i
2bR 2bR E 3
Current induced I= = =0.6 B
8 5
E
Sol 6 : Let terminal velocity be V Voltage induced in each side V1=
3
E=– Blv
V1=1 V
b V
I=– Now each side acts like a battery with a resists
R
B2  2 v
Force due to magnetic field fi=iLB=–
R
\VAB=V1=iRAB =1 – 0.6 (1)=0.4 V
Force due to gravity (f2)=mg
f1 + f2=0 dB
Sol 9 : =– 3 × 10–4 (here is taken positive)
dt
B2  2 v Rmg AdB
⇒ mg – =0 ⇒ v= E=–
R B2  2 dt
2 2 . 6 8 | Electromagnetic Induction and Electromagnetic Waves

E=– 10–2 × (– 3 × 10–4)=3 µ V F qr  dB  1


α= =  .
It is induced clock wise. mr 2  dt  mr
Note: It you get confused with direction, remember
q  dB 
the induced emf produces current, which produces ⇒ α=  
magnetic field. This field will be opposite to direction of 2m  dt 
change. i.e, if B1=B – ∆B1, then induced B will produces q
∆B2 such that it opposite sign of ∆B1. ⇒ α=
2m
( 0.2t )
Sol 10 : The electron experiences the force tangentially, dω q
⇒ = (0.2t)
along the circular paths of induced emf. dt 2m
dB t
⇒ E=pr2. 0.1q 0.1q t2 0.1 × 2 × 102
dt
a=
m ∫ t.d t = =
m 2 50 × 10 –3 × 2
=200 rad/sec.
0
E=pr2K
Sol 13 : Let perpendicular distance of bar from vertex
E=2pr.E
be x
F =qE x=vt
F 1
Acceleration a= Area of triangle A= x (2x) =x2
m 2
qE A=V2t2
a=
M (i) These φ=BA
E rK
⇒E= = φ(t)=BV2t2=0.35 × (5.2)2t2
2πr 2
φ(3)=9.464 (3)2 =85.22 Tm2
q q rK erK
a= E= = dφ
m m 2 2m (ii) emf e=–
dt
(charge of electron is e)
C(t)=– 2BV2t=– 18.93 t
dB
Sol 11 : e=– A e(3)=– 18.93 (3)=56.8 V
dt
dQ dB |e(3)|=56.8V
R =– A
dt dt (iii) e(t)=– 2BV2t
–A  πa2  E(t)=Kt
⇒DQ = ∆B= –  (–c) 
R  R  It varies linearly
 
2
πa C
DQ= πr 2
R Sol 14 : (i) A=
4
dB dB
Sol 12 : e=pr2. =3 × 10–4
dt dt
dB πr 2 dB π
E πr 2 dB e=– A = = × (0.1)2. 3 × 10–4
Ε= = . dt 4 dt 4
2πr 2πr dt
e=2.4 × 10–5 V
r  dB  Induced emf is 2.4 × 10–5V
e=  
2  dt 
(ii)It flows from c to b, to reduce the increasing emf.
qr  dB 
F=qE=   (F is tangential at every point)
2  dt  Sol 15 : (i) f=ilb

F=Ia 10 × 103=i × 3 × 10 × 10–6
F . r=mr2 .a i=3.3×108A
P hysi cs | 22.69

(ii) P=i2R e=– Blv 0< x < w=0 w < x < 3w=Blvw < x < 4w
P 2
=i = 4.1 × 107 W
R Sol 17 : ∫ B.ds =m0ienc
(iii) Totally unrealistic
B.2pr=m0im in cos wt
 w µ0Nim cos ωt
Sol 16 : φ=Blvt 0 < t <  B=
 v 2πr
(downwards positive) dφ=B.dA=B. h.dr
w 3w  µ0N imcos ωt
=Blw  <t<  dr
v v  dφ= h
2π r
 4w   3w 4w  µ0N imcos ωt
=Blv  – t   <t<  b
 v v  ⇒ φ= h/n
 v  2π a

e= –dφ wµ0N imhsin ωt b
dt ∈= = ln
dt 2π a
 w
⇒ e=– Blv 0 < t < 
 v µ0N ωhim b
Amplitude= ln
2π a
w 3w 
=0  <t< 
v v 
Sol 18 :
 3ω 3ω 
=Blv  <t< 
 v v 
∈→
F=iB= B
R
– 2B2 V  w
⇒ E= 0 < t <  dB
R  v e= A
dt
w 3w  dB
= 0  <t<  ⇒ e1= A1
v v  dt
dB
– 2B2 v  3w 4w  ⇒ e2= A2
=  <t<  dt
R  v v 
⇒ e1 is along CD and ⇒ 2 along DC
(here=– )
dB
x=vt \e (A1 – A2) along CD
dt

 2 2
A1 – A2= R –
2 4

 2 2 dB
\e= R –
2 4 dt

Sol 19 : Take loop AEFD


DB
⇒ e1=A1. =1 × 1=1V
DT
Take loop EBCT
DB 1
⇒ e2= A2 = × 1 = 0.5 V
Dt 2
2 2 . 7 0 | Electromagnetic Induction and Electromagnetic Waves

Lets use superposition of current

(ii)

 –Rt 
I= I0 1 – e L 

 
 

I0=steady state current


2
e B ωr
= =
R 2R

B ωr 2  
–Rt
I= 1 – e L 
2R  
 
dTm=rdE
iAE=i11 + i21iEF=i13 – i23iBE= i12 +i22
(Tm=torque due to magnetic field)
e1 3
i11= = A dFm=Bidl
1 11
3+
1 Bidr
1+
2
(tm=magnetic force)
2 2
i13= i11= A
2+1 11 d.Tm=Birdr
1 Bir 2
i12=i11 – i13= A ⇒ Tm=
11 2
e2 2
ωB 2 g 4  – 
Rt
i22= = A
1 11 ⇒ Tm= 1 – e L 
2+ 4R  
1  
1+
3 fg=mg cos θ (fg=force of graving)
3 3
i23= i22= A fgr  r  mgr cos(θ)
3+1 22 Tg= = mg cos θ   =
2 2 2
1
i21=i22 – i23= A
22 mgr cos θ ωB2r 4  – 
Rt
\ T= + 1 – e L 
3 1 7 2 4R  
 
iAE= + = A
11 22 22
Sol 21 : e=BLV(V is terminal velocity)
2 3 1
iEf= – = A
11 22 22 e BLV
i= =
R R
1 2 3
iEB= + = A Fm=iLB (fm=force due to magnetic field)
11 11 11
B2L2 V
=
Sol 20 : (i)d∈=Bvdr R

de=Brωdr Fg=mg (fg=force due to grouts)


2
B ωr
e= B2L2 V
2 mg=
R
B ωr 2
Emfacross the terminals of switch is
2
P hysi cs | 22.71

mgR Integrating on both sides.


⇒ v= 2 2
BL B0 av
g–
–mR mR =t
B2L2  V  mg ln
2 2 2 2
(ii) Fm=  = B0 a B a v
R 2 2 g– 0 0
mR
fg=mg ⇒ v0=0 (initially dropped fromrest)
mg mg
f=fg – fm=mg – = B20 a2 v
2 2 g– 2 2
ln mR =– B0 a t
F=ma g mR
g
\a=  –B2a2t 
2 B20 a2 v
 0

g ⇒ = g  1 – e mR 
\acceleration of the mass is mR  
2  
B0 y  –B2 2 
(iii) (a) dφ=B.dy = . a. dy oa t
mgR  
a ⇒ v= 1 – e mR 
B
(
φ= y 22 – y12
2
) B20 a2 


(y1, y2 are instantaneous heights of edges parallel to


µ0i
x-axis) Sol 22 : (a) B=
2πr
B0 a
φ= (y 2 + y1 ) ( y2 – y1=a) Let magnetic field due to upward current be B,
2
µ0 I
dφ B0 d d B 1=
= (y + y1) 2πr
dt 2 dt 2
Force due to it be φ,
B0 a dy dy1 dy 2 µ0 I
= (2V) (v= = = ) dφ,= .a.dr
2 dt dt dt 2πr
dφ µ0 Ia 2a
=B0aV f 1= ln
dt 2π a
ε dφ 1 B0 aV µ0 Ia
i= = . = f1= ln 2
R dt R R 2πr
(b) fm=SiB (fn is magnetic force) µ0 Ia
Similarly f2= ln 2 (f2 is by downward current)

B y B y  B av  B 
= i a  0 2 – 0 1  = 0 .a  0 (y 2 – y1 ) \φ=f1 + f2=m0Ialn 2
 a a  R a 

B20 a2 v µ0 I0 a.ln2sin ωt
F m= φ=
R π

dφ µ ωI aln2
fg – fm e=– =– 0 0 cos wt
(c)a= dt π
m
Q=CV
B20 a2 v
mg – dQ dv (µ ω2 I aln2)
dv R i= =C =C 0 0 sin wt
= dt dt π
dt m
µ0 a
dv Imore= CI0w2 ln2
=at π
B20 a2 v – µ0 ωI0 acln2
g– (b) Q=CV= cos wt
mR π
2 2 . 7 2 | Electromagnetic Induction and Electromagnetic Waves

Sol 3 : (D) Work done is zero as magnetic fields is


uniform

Sol 4 : (A) Let voltage induced be V.


eN
Total charge a=
m
V
Electric field E=
µ0 ωI0 a.cln2 L
Q0=
π QE=|f1 – f2|

Sol 23 : B=m0 in eM V
. =|f1 – f2|
M L
φ=B.A=m0 in pa2
V=|f1 – f2| ML/eM
φ=m0 npa2 i0 sin wt
–dφ Sol 5 : (B) Here power Supplied=Heat generated as no
∈= =– m0npa2ωI0 cos wt
dt other element is using I,
e. e.2πR Q
Resistance of shell, r5= = ⇒F . V=Q ⇒ F=
A L.d V
∈ Sol 6 : (A) area of loop A=a2
i=
rs
dA da
= 2a = 2a (2V)= 4av

∴I=
(µ ni ω cos ωt)πa (Ld))
0 0
2 dt dt
dA
ρ.2πR B =iR
dt
B
Exercise 2 ⇒ i=
R
=(4aV)

⇒ R=4ar
Sol 1 : (B) Let angular velocity be ω.
Bv
ωe ⇒ i=
i=– r

µ0i – µ 0 ωe Sol 7 : (B) Let height of triangle be a at time t, area


B= = inside the magnetic field
2R 4 πR
1 2 (a – vt)2
φ=B.A=
– µ 0 ωe
. pr =
2
– µ0 ωer 2 A=
2
( a – vt ) ( a – vt ) =
πR 4R 3 3
2 2 dA
–dφ µ0 er dw µ0 er α ∈=– B
∈= = = dt
dt 4R dt 4R
d (a – vt)2
iR=– B
dA BdA dt 3
Sol 2 : (C) e= B
⇒ iR=
dt dt
B 2
dQ BdA B i= . (a – vt).v
R. = ⇒ dQ= dA R 3
dt dt R
2BV 2BVa 2BV 2
B i= (a – vt)= –
⇒DQ= DA R 3 R 3 R 3
R
i=C1 – C2t
DA=2A (as it is rotated by 180º)
2AB
∴DQ=
R
P hysi cs | 22.73

Sol 8 : (B) A=A0 cosq Sol 13 : (A) a=Blv


θ=wt Q
=– BLv
φ=nBA c
Q=BlVC=4 × 1 × 20 × 10 × 10–6=800 µC
φ=nBA0 cos wt
Phas greater potential than Q as
–dφ
e= =nωBA0 sin wt
dt [V × B] is directed towards P
e nωBA0 Hence qA is the
i= = sin wt
R R \qA=+800 mc qB=– 800 mc
nωBA0
Amplitude=
R Sol 14 : (D) it is independent or resistance
ω=100 (2π)=200 p
e=qvB
⇒Amplitude
e=q.rωB
100 × 200π × 10 × 10 –3 × π × (10 –1 )2
= = 2A
10  – 
Rt
Sol 15 : (B, D) i= i0  1 – e L 
di  
Sol 9 : (A) e= –L  
dt V
i 0=
⇒ – ∫ e.dt=Li
∈ .dt R

d i0 is same,
⇒ e= – BAN
dt ⇒ R1=R2

∫ ∈e at
dt=BAN
Time constant t=
L
R
⇒ Li=BAN
tC>tB
i=BAN/L
⇒ L2> L1
1
Sol 10 : (B) B ∝ φ φ t ∈
L Sol 16 : (B) = . = × t=it=Q
R t R R
A ∝ l2 Here charge (B)
2
∴L ∝
L Sol 17 : (A, B, D) For both P, Q it is induced inward
hence clockwise.
Sol 11 : (A) Induced current is along DC for loop DC. iR=0 which is obvious
For loop AB it should be along AB but since area of CD
loop is greater than AB loop, hence current is along BA.
Sol 18 : (A) If i increases B increases, to reduce B, they
(A) repel

–dB
e=
dt
( ACD – A AB ) Sol 19 : (B, C) Antilockwise means field should increase
into plane.
∴A cos DC
Sol 20 : (A, B) Magnetic force fm=iB
Sol 12 : (B, D) Opposite currents (anti parallel currents)
Gravity force fg=mg
repel
fm cos θ=mg sin q
Hence (D)
ilB=mg Tan q
I2 induces opposite current to oppose the increase flux
is show from E to F.
(B)
2 2 . 7 4 | Electromagnetic Induction and Electromagnetic Waves

Sol 21 : (B) fm would be along rats \f1=(L2 + l2)B


fm=mg sin q Case II : A=L2 – l2
∴B=mg sin q \d=(L2 – l2)B
Sol 31 : (C) Clockwise current is induced
Sol 22 : (A, B, C, D)
Current from  to c and b to a
B is along the plane of ring Hence, it cannot t be
induced, irrespective of directions of motion
Sol 32 : (B) Clockwise overall current

Sol 23 : (A, C) \f toe, b to a

Its common knowledge Reason will be taught in higher


Sol 33 : (B) f2<f1
classes.
∴I2< I1
Sol 24 : (D) Induced current is anti-clockwise hence i2
along dc, i, along ab
Sol 34 : (A) Di-electric ring which is uniformly charged
i1=i1 since there are in same wire has stationary any charges. Hence time independent
electrostatic field out of system
Sol 25 : (B, D) Assume mA=mB A→P
Then iA>iB (hA>hB) (B) Rotating charge produce magnetic field within the
⇒ PA> PB system, and hence induced electric field. But outside
remains unchanged as it is
Now is mA<mB and PA> PB then surely hA>hB
Di-electric

Sol 26 : (C) Opposite current will induce in the upper B → P, Q, S


ring and it will get repelled by the coil at the bottom (C) Current in a ring produces magnetic field hence
induced electric field
Sol 27 : (B, D) C → Q, S
V α horizontal displacement (D) Current carrying ring has magnetic field and
\VQP=VPO, VPO=VRO induced electric field.
\VQ= – VP=VP – VO, VP=VR>V0 µ=Ipr2
µ=pr2I0 cos wt
Sol 28 : (A, C) d∈=Bvdr
µ charges with time,
de=Bω rdr
⇒ magnetic moment charge Q, R, S
B ωr 2
⇒ e=
2
Bωa2 Previous Years’ Questions
VP–V0=
2
Sol 1 : (C) In uniform magnetic field, change in magnetic
Bw(2a)2
VQ – V0= =2Bwa2 flux is zero. Therefore, induced current will be zero.
2
\correct answer is (c)
Vp – VR=0

Sol 2 : (D)
Sol 29 : (D) No current flows. As it doesn’t form a
closed circuit. Cross × magnetic field passing from the closed loop is
increasing. Therefore, from Lenz’s law induced current
Sol 30 : (C) φ= ∫ B.dA will produce dot • magnetic field. Hence, induced
current is anticlockwise.
Case I : A=L2 + l2
P hysi cs | 22.75

Sol 3 : (C) Sol 6 : (D)


From conservation of energy,
1 2 1
LI = CV2
2 max 2
C
∴ Imax=V
L

L=2pR Sol 7 : (C)


L Comparing the LC oscillations with normal SHM, we get
\R=
2π d2Q
=– w2Q
2T sin(dθ)=Fm dt 2

From small angles, sin(dθ)=dq 1 d2Q


Here, ω2= ∴ Q=– LC
LC dt2
\2T(dθ)=I(dL)B sin 90°= I(2R. dθ) . B
ILB Sol 8 : (A, C, D) From Faraday’s law, the induced voltage
∴ T=IRB=

 di 
∴Correct option is (c) V ∝ L, if rate of change of current is constant  V = –L 
 dt 
V2 L2 2 1 V1
Sol 4 : A → r, s, t ; B → q, r, s, t ; ∴ = = = or =4
V1 L1 8 4 V2
C → q, p ; D → q, r, s, t
Power given to the two coils is same, i.e.
In circuit (p) : I can’t be non-zero in steady state.
i1 V2 1
In circuit (q) V1i1=V2i2 or = =
i2 V1 4
V1=0 and V2=2I=V (also)
1
In circuit (r): V1=XLI=(2pfL) I Energy stored W= Li2
2
= (2π × 50 × 6 × 10–3) I=1.88 I 2
W2 L   i2  1 W1 1
V2=2I ∴ =  2    =   (4)2or =
W1  L1   i1  4 W2 4
In circuit (s): V1=XLI =1.88 I

 1   1  Sol 9 : (B, C)
V2=XCI=  I =  l=(1061) I
2 πfC –3 
   2π × 50 × 3 × 10  2
 1 
Z= R 2 + X 2C = R 2 +  
In circuit (t):  ωC 
V1=IR=(1000) I In case (b) capacitance C will be more. Therefore,
V2=XCI=(1061) I impedance Z will be less. Hence, current will be more.
Therefore the correct options are as under ∴ Option (B) is correct
(A) → r, s, t ; (B) → q, r, s, t ; Further,
(C) → q, p; (D) → q, r, s, t VC= V 2 – VR2 = V 2 – (IR)2

Sol 5 : (B) In case (b), since current I is more.


Charge on capacitor at time t is Therefore, VC will be less.
q=q0(1 – e–t/ τ ) ∴ Option (C) is correct
Here, q0=CV and t=2t ∴Correct options are (B) and (C)
∴ q=CV(1 – e–2τ / τ )=CV(1 – e–2)
2 2 . 7 6 | Electromagnetic Induction and Electromagnetic Waves

Sol 10 : (7) Sol 15 : (8)


Assume circular wire loop as primary and square loop At t=0, current will flow only in 12 Ω resistance
as secondary coil 5
∴ Imin =
2µ0iR 2 12
=
φsec ondary × a2 × cos 45o
2(3R 2 + R 2 )3 / 2 At t → ∞ both L1 and L2 behave as conducting wires

µ0iR 2 2 3
= × a2 × ∴ R e ff =
2 × 8R 3 2 2

10
φsec ondary µ0 a2 Imax =
=M = 3
i 23 × 21/2 R
Imax
µ0 a2 =8
M= Imin
7/2
2 R
1mH 3

Sol 11 : (A, C)
Total flux associate with loop=0 2mH 4
Therefore emf=0 in any case.

Sol 12 : (B) 12


For direct transmission
P = i2R = (150)2 (0.4 × 20) = 1.8 × 105 W
5V
1.8 × 105
Fraction (in =
%) × 100
= 30 %
6 × 105 S

40000 Sol 16 : (C, D)


Sol 13 : (A) = 200
200
For right edge of loop from x=0 to x=L
Sol 14 : (A, D) vBL
i= +
R
o o
45 45
vB2L2
h=10 cm 1 F iLB
= = (leftwards)
dr R

dv vB2L2
−mv =
dx R
h
µ0 I µ0 Ih B2L2
∴ v(x) =
v0 − x
=
φ w ∫ =
2πr
2rdr
π mR
0
v 0BL B3L3
i(x)
= − x
µ 0h R mR 2
So, Mutual inductance Mw =
π
v 0B2L2 B 4L4
µ0h di µ0 F(x)
= − x (leftwards)
∴ =
εw = R mR 2
π dt π
Due to rotation there is no change in flux through the
wire, so there is no extra induced emf in the wire. From
Lenz’s Law, current in the wire is rightward so repulsive
force acts between the wire and loop.

S-ar putea să vă placă și